Civil Procedure Law Outline

Personal Jurisdiction – Jurisdiction over People and Things (Property)

IN PERSONAM JURISDICTION
Judgment has power over the person. Can be satisfied wherever someone finds you. For a judgment to be in personam, that person has to be present. In Personam cases can only be served in state. They must be served while present in the state. Physical presence is required.

To get In Personam Jurisdiction you need
1) Presence
2) Consent
3) Domicile
4) An act outside the state but within the state’s long-arm statute

Note: (COLLATERAL ATTACK): Say you are called to an out of state court, but don’t believe they have jurisdiction. By failing to appear, the court will usually automatically side for the plaintiff. When the person comes to your state in suit to enforce the judgment you can raise the issue of a lack of jurisdiction. If you lose, you’re sunk and you cannot whatsoever defend on the merits. The only opportunity to do so was in the first suit.

IN PERSONAM JUDGMENTS
Was the judgment in Mitchell v. Neff a valid in rem judgment?
In Rem Judgment – 3 types
1) True in rem – a judgment that affects the property and is good against the whole world. For Example: registering a title to land
2) Quasi-in-rem (Type I) – a dispute about particular property and the interests of particular persons in that property. For Example: The partitioning of Real Estate or the interests of people in a Trust Fund.
3) Quasi-in-rem (Type II) – A proceeding where property is used to give the court power to exercise authority over a dispute that’s really not about the property. That property is in the state, is attached, and is brought before the court. “Here’s some property the defendant has in the state. If I win you can use this property to satisfy the judgment.” (Established by Harris v. Balk – A owes B and B owes C. A goes to MD and C sues him there. A goes back home and B then sues him too)

Pennoyer v. Neff
FACTS: Neff’s claim to the property is that his land was wrongfully sold to Mitchell by Pennoyer. Pennoyer’s claim is that the Sheriff sold him the land legally. Neff never even had possession of the land before it was taken possession by Pennoyer.
REASONING: Neff was out of state, out in California. He was not present in the forum. He was also not personally served with the affidavit to appear in court.

In Mitchell v. Neff – Neff never had the title or deed when the judgment was rendered. Technically the defendant never had ownership of the property. There should have been nothing for the court to give to Mitchell for the judgment. There was no valid Quasi-in-rem judgment.

Hess v. Pawloski
FACTS: The two are involved in an automobile accident in Massachusetts when Hess strikes Pawloski. Hess is a resident of Pennsylvania. He is ordered to go to a Massachusetts court to try the case, but how?
REASONING: "Non-resident motorist statute" - If you come into our highways, drive on our highways, you get into an accident, you have then consented to the appointment of our registrar. Service of process will be given to our registrar, and you will receive notice of the suit and an opportunity to defend. We can serve you outside the state. The person doesn't always have to be physically present to be served. CONSENT.

Special Appearance – If you are contesting personal jurisdiction in a court you make a special appearance to that court in order to present your claim. In a state court, any further defense on merit will establish consent to jurisdiction. In a federal court, there are some actions allowed to be filed in Rule 12 (b)(2)

International Shoe v. Washington
FACTS: International Shoe employed salesmen under direct control of sales managers located in St. Louis. These salesmen resided and sold shoes to customers only in Washington. The state of Washington has an unemployment compensation fund that requires employers to contribute money to the fund. A commissioner issued an order and notice of assessment of delinquent contributions from 1937 to 1940 to a sales solicitor of the company as well as mailing a copy to the head office in St. Louis, Missouri. International Shoe claims that the service upon the salesmen was not proper and that the corporation is not doing any business within the state.
REASONING: The extent that a corporation exercises the privilege of conducting activities within a state, it enjoys the benefits and protection of the laws of that state. The activities carried on in behalf of Shoe were neither irregular nor casual. They resulted in a large volume of interstate business, in the course of which the appellant received the benefits and protection of the laws of the state, including the right to resort to the courts for the enforcement of its rights. There is no unreasonable or undue procedure. Due process requires only that in order to subject a defendant to a judgment in personam, if he be not present within the territory of the forum, he have certain minimum contacts with it such that the maintenance of the suit does not offend 'traditional notions of fair play and substantial justice.'

Due Process Requirement: In order to subject a defendant to a judgment in personam, if he be not present in the forum, he must have certain minimum contacts with it such that the maintenance of the suit does not offend "traditional notions of fair play and substantial justice."

Categories from International Shoe case:
1. Clear jurisdiction (or none at all)
· Substantial, systematic, continuous activity in the state, C/A comes from that activity
· Isolated, sporadic, casual activity, C/A unrelated to that activity
2. Unclear
· General In Personam Jurisdiction - Substantial, systematic, continuous activity, but no C/A from that activity
· Specific Jurisdiction - Activity is sporadic, isolated, not continuous, but the C/A comes from this activity (One contact is enough)
Int'l Life Insurance 1957
Hess v. Pawloaski. 1927

McGee v. International Life Insurance Co.
FACTS: In 1944, Lowell Franklin, a resident of California purchased life insurance from an Arizona corporation. In 1948, International Life Insurance Company of Texas bought the Arizona corporation and with Franklin's permission assumed the insurance obligation. Franklin paid premiums from his California home until his death in 1950. Franklins mother, the beneficiary sent proofs of his death, however, the insurance company refused to pay claiming suicide. Neither the Arizona nor the Texas company ever had any office or other business in California besides this policy.
REASONING: Due process requires only that in order to subject a defendant to a judgment in personam, if he be not present within the territory of the forum, he have certain minimum contacts with it such that the maintenance of the suit does not offend 'traditional notions of fair play and substantial justice.' The suit was based on a contract which had substantial connection with that state. The respondent had adequate notice of the suit and sufficient time to prepare its defenses and appear.
NOTE: One contact is enough for specific jurisdiction so long as the cause of action arises out of that contact and the contact is substantial in its “nature and quality.”

World-Wide Volkswagen Corp. v. Woodson
FACTS: The Robinsons purchased a car from the plaintiff, a car dealer in New York. While driving this car through Oklahoma, the Robinsons were rear-ended by a third party. The crash caused a fire which severely burned Kay Robinson and her two children. The Robinsons brought a product liability in Oklahoma. Volkswagen is incorporated and has its business office in New York. There is no evidence that the defendant does any business in Oklahoma, ships or sells any products to or in that state, has an agent to receive process there, or purchases advertisements in any media calculated to reach Oklahoma.
RESONING: The concept of minimum contacts protects the defendant against the burdens of litigating in a distant or inconvenient forum. It acts to ensure that the states through their courts, do not reach out beyond the limits imposed on them by their status as coequal sovereigns in a federal system. Volkswagen carries no activity whatsoever in Oklahoma. They close no sales and perform no services there. They avail themselves of none of the privileges and benefits of Oklahoma law. Thus, the Due Process Clause does not contemplate that a state may make a binding judgment in personam against an individual or corporate defendant with which the state has no contacts, ties or relations.

Two functions served by minimum contacts test.
1. Protects defendant against burdens of litigating in distant or inconvenient forum; insures fairness, reasonableness
2. Serves to Protect State Sovereignty


Keeton v. Hustler Magazine
FACTS: Keeton is a resident of New York, her only connection to New Hampshire is that the magazine she assists in producing circulates in that state. Plaintiff sues for libel. The magazine, Hustler, is based in California. New Hampshire was chosen as the forum because their statute of limitations is 6 years for a libel lawsuit, any other state wouldn't allow the lawsuit due to the statute.
REASONING: Even though only a small portion of copies of the magazine were distributed in New Hampshire, the tort of libel is generally held to occur wherever the offending material is circulated. Regular monthly sales of thousands of magazines cannot by any stretch of the imagination be characterized as random, isolated, or fortuitous. The plaintiff is not required to have "minimum contacts" with the forum State before permitting that State to assert personal jurisdiction over a nonresident defendant

Calder v. Jones
FACTS: Shirley Jones brought a suit in California Superior Court claiming that she had been libeled in an article written and edited by National Enquirer based in Florida. The magazine which the defendants work for has a strong circulation in California, 600,000 copies which is twice as much as the next state, New York. Both defendants are Florida residents who rarely, if ever, visit California, and have never done so to work on the story involving the respondent.
REASONING: A valid basis for jurisdiction existed on the theory that petitioners intended to, and did, cause tortious injury to the respondent in California. California is the focal point of both the story and of the harm suffered. The tortious and intentional actions were aimed at California. Under the circumstances, they knew that the injury would be felt in the State in which the respondent lives and works and in which the National Enquirer has its largest circulation. An individual injured in California need not go to Florida to seek redress from persons who, though remaining in Florida knowingly cause the injury in California. California was the targeted state for the tortious act and therefore is enough to satisfy personal jurisdiction under the EFFECTS TEST. EFFECTS TEST IS NOW THE STANDARD FOR TORTS AND JURISDICTION.

Hanson v. Denckla
HOLDING: If you want to exercise jurisdiction over a non-resident defendant, at least that defendant must have done something purposefully availing itself of the privilege of conducting activities within the forum state, invoking the benefits and protections of its laws. Purposefully availing yourself means: Going into a state with intention to do business. You will benefit from the protection of laws.


Now, in order to comply with personal jurisdiction, the court must comply with:
1) State Requirements – Long Arm Statute
2) Federal Requirements – Minimum Contacts (Foreseeability, Purposeful Availment) Gestalt Factors, Relatedness,

I. MINIMUM CONTACTS

1. Depends on the “quality nad nature”
2. Purposefulness (A corporation that chooses to conduct business in a state accepts a reciprocal duty to answer for its in-state activities in the local courts.)
3. Magnitude (Dollar value of deals or wrong committed)
4. It is foreseeable that you can be called into court.
5. Related to the cause of action?

FORESEEABILITY

Foreseeability requires defendant's conduct and connection w/ forum state must be such that defendant should reasonably anticipate being haled into court there.

PURPOSEFUL AVAILMENT

Seeking to benefit from the laws and activities in the state

ANTICIPATION

A person with these minimum contacts must reasonably anticipate being haled into court.

II. RELATEDNESS

The cause of action must arise from the contacts with a state or the contacts must be continuous and substantial such that a person or company purposefully avails itself of the privileges of that state.

III. GESTALT FACTORS (FAIRNESS TEST)

Defendant does not incur an unfair burden in dispute
Difficult to prove, It must be gravely difficult and inconvenient
Forum state's interest in adjudicating dispute
Plaintiff's interest in getting convenient, effective relief
Interstate judicial system's interest in efficient resolution of controversy
Shared interest of several states in furthering substantive policies
Last two interests are not usually playing much of a role, but they're there



Ø SPECIFIC JURISDICTION
Activity is sporadic, isolated, not continuous, but the C/A comes from this activity (One contact is enough)
Int'l Life Insurance 1957
Hess v. Pawloaski. 1927
Requires the state long arm statute
Requires Due Process (Relatedness, Gestalt Factors, Minimum Contacs)


Burger King Corp. v. Rudzewicz
FACTS: The defendant entered into a business relationship to receive a Burger King franchise in Michigan. A preliminary agreement was formed such that the defendant would assume operation of an already existing facility while his business partner attended management courses in Miami where Burger King is headquartered. The parties had many disagreements. By signing final agreements, Rudzewicz obligated himself to payments over $1 million over 20 years. Soon after the defendant fell behind in monthly payments to the Miami headquarters. Burger King commenced instant action invoking the Florida court's diversity jurisdiction and its original jurisdiction over federal trademark. Burger King has all contracts provide that the franchise relationship is established in Miami and governed by Florida law, and call for payment of all required fees and forwarding of all relevant notices to the Miami headquarters.
REASONING: Rudzewicz established a substantial and continuing relationship with Burger King's Miami headquarters, received fair notice from the contract documents and the course of dealing that he might be subject to suit in Florida, and has failed to demonstrate how jurisdiction in that forum would otherwise be fundamentally unfair. Rudzewicz deliberately "reached out beyond" Michigan and negotiated with a Florida corporation for the purchase of a long-term franchise and the manifold benefits that would derive from affiliation with a nationwide organization. His relationship to the company in Florida can in no sense be viewed as "random," "fortuitous," or "attenuated." Agreeing to settle any disputes under Florida law is sufficient for foreseeability. Rudzewicz's refusal to make the contractually required payments in Miami and his continued use of Burger King's trademarks and confidential business information after his termination caused foreseeable injuries to the corporation in Florida.

Asahi Metal Industry Co. v. Superior Court
FACTS: The petitioner manufactures tire valve assemblies in Japan and sells them to several tire manufacturers. The sales took place in Taiwan where Cheng Shin is located. The valve assemblies are assembled in Japan and shipped to the United States. Twenty Percent go to California. A motorcycle accident occurs in California which causes the death of a passenger on the motorcycle due to a defective tire. A product liability suit was brought to California Superior Court caused by a defect in a tire manufactured by a manufacturer. Cheng Shin (manufacturer) filed a cross-complaint against Asahi. The victim then settles with Cheng Shin so the only case left is the cross-claim between Cheng Shin and Asahi.
REASONING: The burden imposed on petitioner by the exercise of state court jurisdiction would be severe, since petitioner would be required not only to traverse the distance between Japan and California, but also to submit its dispute with Chang Shin to a foreign judicial system. The interests of Cheng Shin and the forum State in the exercise of jurisdiction over petitioner would be slight, and would be insufficient to justify the heavy burdens placed on petitioner. The facts do not establish minimum contacts sufficient to render the State's exercise of personal jurisdiction consistent with fair play and substantial justice as required by the Due Process Clause. Had Asahi known or purposely directed a substantial portion of his products to California, that would be sufficient.

O’Connor Analysis (Scalia, Rehnquist, Powell)
Purposefully directed - Asahi did not purposefully direct the valves to California. Not just place it there and let it fall into the stream. Something must be done more than putting a product into commerce. Intent to benefit from the product going to California. There needs to be an action by the defendant purposefully directed at California. Mere awareness is not enough.

Brennan Analysis
Stream of Commerce is not unpredictable. Currents/eddies. There was a regular, anticipated flow from Mfr to distributor to retailer. Asahi knew that the distributor made regular sales to California and this should be enough to establish the purposeful direction requirement



Ø GENERAL JURISDICTION
Substantial, systematic, continuous activity, but no C/A from that activity
More difficult to prove than Specific Jurisdiction

Helicopteros Nacionales v. Hall
FACTS: A helicopter owned by Helicol crashed in Peru. Four United States citizens died. The decedents were employed by Consorcio, a Peruvian consortium and were working on a pipeline in Peru but had its headquarters in Houston, Texas. The Peruvian company was formed to enable the venturers to enter into a contract with a state owned oil company. The CEO of Helicol flew to Houston and eventually a contract was signed for transportation. It was written in Spanish on official government stationery. The contract also provided that the residence of all parties would be Lima, Peru. All controversies arising out of the contract would be under the jurisdiction of Peruvian courts. Aside from the negotiation session in Houston, Helicol had no other contacts with Texas. It had purchased its fleet from Bell Helicopter in Fort Worth and sent prospective pilots to Fort Worth for training. Helicol has never been authorized to do business in Texas and never has had an agent for the service of process within the State.
REASONING: It is undisputed that Helicol does not have a place of business in Texas and never has been licensed to do business in the State. The contacts consist of sending the CEO to Houston for a contract-negotiation session; accepting into its New York bank account checks drawn on a Houston bank; purchasing helicopters, equipment and training services from Bell Helicopter for substantial sums; and sending personnel to Bell's facilities in Fort Worth for training. The one trip by the CEO cannot be described or regarded as a contract of a "continuous and systematic" nature. The Texas Supreme Court focuses on purchases and the related training trips. Previous cases make clear that purchases and related trips are not a sufficient bases for the assertion of jurisdiction. Mere purchases even if occurring at regular intervals, are not enough to warrant a State's assertion of in personam jurisdiction over a nonresident corporation in a cause of action not related to those purchase transactions.

What was missing: Some kind of permanent employees, offices, agents. Something continuous, automatic, substantial. Something we can equate with PRESENCE - under Pennoyer



Ø FIRST CIRCUIT, SJC CASES

Marino v. Hyatt (1st CIRCUIT)- 1986
FACTS: Plaintiffs are Massachusetts residents and defendant Hyatt is a Delaware corporation with its principal place of business in Chicago, Illinois. Plaintiffs made reservations and contracted to stay at the Hyatt in Maui, Hawaii through the Rite Way Travel Agency in Methuen, Massachusetts. Plaintiffs traveled to Maui and took up lodging in the Hyatt Regency. During the trip, Mary Marino was injured when she slipped and fell in the bathtub of the hotel of the room she shared with her husband.
REASONING: The cause of action must "arise from" the defendant's transacting business in Massachusetts. It must be shown that the plaintiffs' personal injury claims, arose from one or more of Hyatt's business transactions within Massachusetts. Court says that when you make a reservation, there is no covenant, only that there is going to be a room for you. The covenant begins when you are handed a room key. Slip and Fall was a tort claim, and a reservation is not a guarantee that there would be safety and security. C/A arose from Hawaii.

This is a federal court construing a state long-arm statute. Under the state long arm, we don't think a but for connection between the c/a and instate activity is enough. The but for connection does not make the c/a arise out of instate activity.

Tatro v. Manor Care, Inc. (SUPREME COURT OF MASS)- 1994
FACTS: The plaintiff, Claire Tatro, brought this personal injury action against the defendant, Manor Care, Inc., a Delaware corporation doing business in Anaheim, California. While attending a conference at the hotel, she was injured when she slipped and fell in the bathtub in her room. The conference was for the Council of Hypnotist Examiners. The hotel made plans with the members by sending out pre-printed registration cards to those who registered in advance. The second mailing then specified that late registrants should speak with the hotel directly. The plaintiffs were late registrants. They reserved a room at the hotel by the telephone. The defendant did not advertise in Massachusetts from 1987-9, however, they do have a number of contacts with Massachusetts residents. Many Mass. Corporations hold conferences at this hotel.
REASONING: It is obvious that the defendant transacted business in the Commonwealth. The defendant's acceptance of the plaintiff's room reservation formed part of the defendant's overall purposeful solicitation of hotel business from residents of Massachusetts. The defendant's reservation contract with the plaintiff was one part of a broader range of activities that, literally, amounted to the transaction of business in Massachusetts. The plaintiff further contends that her injury "arises from" the defendant's transaction of business in Massachusetts, because, "but for the hotel's solicitation of her business and subsequent contract with her for the hotel room in which she was injured, her injuries would not have occurred." In view of the hotel's quite extensive solicitation of business in Massachusetts, it is reasonable to conclude that the hotel has "invoked the benefits and protections" of Massachusetts laws. It is not unreasonable to require a hotel that deliberately draws its customers from many if not all of the states.

Nowak v. Tak How Investments, Ltd. (1st CIRCUIT)1996
FACTS: Tak How is a Hong Kong corporation with its only place of business in Hong Kong. Mr. Nowak was employed by Kiddie Products, Inc., which has its place of business in Avon, Massachusetts. Kiddie Products does extensive business in Hong Kong and Nowak makes two business trips to Hong Kong each year, accompanied by his wife on one of those trips. The hotel gave great deals to the company in exchange for guarantees of certain numbers of reservations each year. Since 1992, Kiddie Products employees have stayed exclusively at the Holiday Inn. In June 1993, the Holiday Inn telecopied Colantuone, the vice-president of the company, a message announcing new corporate rates and other promotional materials. A reservation was then made for September 16 of that year for Mr. Nowak and his wife. Mrs. Nowak drowned in the hotel swimming pool. In 1992 and 1993, prior to Mrs. Nowak's death, Tak How advertised the Holiday Inn in certain national and international publications, some of which circulated in Massachusetts. In addition, in February 1993, Tak How sent direct mail solicitations to approximately 15,000 of its previous guests, including previous guests residing in Massachusetts.
REASONING: When a foreign corporation directly targets residents and achieves its purpose, it may not necessarily be unreasonable to subject that corporation to forum jurisdiction when the efforts lead to a tortious result. The Hotel's solicitation of Kiddie's business and the extensive back-and-forth resulting in the reserving a set of rooms for employees set in motion a chain of reasonably foreseeable events. The possibility that the solicitation would prove successful and that one or more of the guests staying at the Hotel as a result would use the pool was in no sense remote or unpredictable; in fact, the Hotel included the pool as an attraction in its promotional materials. The next matter on purposeful availment focus on voluntariness and foreseeability. Tak How's unprompted June 1993 correspondence with Kiddie Products, which led directly to the ill-fated Hong Kong trip in September was minimally sufficient to satisfy this requirement. Tak How had an obvious financial interest in continuing business with Kiddie Products. In order to be subject to Massachusetts' jurisdiction, a defendant need only have one contact with the forum state, so long as that contact is meaningful. The correspondence rendered foreseeable the possibility of being haled into a Massachusetts court. The advertising in magazines and publications circulating in Massachusetts reveal an even bigger attempt.
The minimum contacts and fairness rule passes all the Gestalt Factors. It is almost always inconvenient and costly for a party to litigate in a foreign jurisdiction. The defendant must demonstrate that "exercise of jurisdiction in the present circumstances is onerous in a special unusual, or other constitutionally significant way."

What to get from these cases:
1) Federal Court (Marino) required proximate cause in relation to the C/A Arising out of transaction in Mass.
2) SJC of Mass required only but for (Tatro)
3) In Nowak, the Federal Court had to follow Tatro because the SJC’s interpretation of the Mass long arm statute can only be overruled by the US Supreme Court.
a. The 1st circuit says they don't agree with interpretation of long arm, but they get their own say under due process (US Constitution based) and believe that proximate cause should be used under due process.
b. Then the court throws in "FLEXIBLE PROXIMATE CAUSE" Strict adherence to proximate cause standard in all circumstances is unduly restrictive.
c. When foreign corporation directly targets residents in ongoing relationship, it may not be unreasonable to subject corporation to forum jurisdiction when tortious result.

Daynard v. Ness, Motley, Loadholt, Etc.
FACTS: Daynard is a law professor at Northeastern Law. The defendant law firms in this case have been litigating against the tobacco industry on behalf of forty-six different states. Daynard says his efforts were central to many of these titanic recoveries. In 1993, Charles Patrick, a partner at Ness Motley came to Boston to meet with Daynard and retain Daynard's services. Throughout his dealings with Ness & Motley and Scruggs & Millette, Daynard understood the two firms to be in a joint venture that at first encompassed the Mississippi tobacco litigation and then broadened to include tobacco litigation nationwide. Daynard traveled to NM's South Carolina offices and from then on continued to communicate regularly by phone and fax with members of NM providing them with advice. Both firms also came to Boston to meet with Daynard. Initially, NM compensated Daynard based on hourly fees for his services. In July and August of 1996, NM and SM both confirmed their agreement to compensate him in the form of a share of the fees. Scruggs promised Daynard 5% of any fees recovered which Daynard accepts and shook on. This was in Chicago in late August, 1996. About a year later while the two law firms began collecting Daynard wrote letters confirming the fee arrangement and identifying certain expenses paid. After there was no response Daynard wrote another letter. A reply was mailed saying that both firms disavowed the 5% fee arrangement. Daynard even had to commit personal funds to the matter by paying $15,000 for a substitute teacher for his teaching obligations. Neither Scruggs nor his firm has ever had and offices, real estate, bank accounts, or other property in Massachusetts. He denies that he or his firm had any role in contacting or retaining Daynard in Massachusetts.
REASONING: This is a case of specific jurisdiction. It is consistent with the Due Process Clause to attribute to the Scruggs defendants the Motley Defendants' retention of and certain interactions with, Daynard where they led Daynard and the public to believe this was a joint venture. The facts alleged by Daynard are sufficient to make the jurisdictional showing that in Boston, Patrick of Ness Motley hired Daynard, that Daynard reasonably understood Patrick to be action on behalf of a joint venture or other agency relationship between NM and SM and that Daynard relied on this understanding by providing his services to both defendants. Further evidence comes from the Chicago meeting where Scruggs said he acted with at least apparent authority for both firms and reached an agreement. The Scruggs defendants had many opportunities to disavow a relationship with Daynard or to clarify the relationship. Scruggs did have some contact with Massachusetts, however, they were minimal. The fax and telephone conversations combined with the visits by firm members and Patrick's physical presence is enough for Scruggs to be "engaged in purposeful activity related to the forum that would make the exercise of jurisdiction fair, just, or reasonable." Finally, the burden on the Scruggs defendants of appearing in Massachusetts, given they routinely represent clients outside their home state, is not by any means unusual. The Scrugg's defendants' contacts properly imputed from the Motley defendants, against the backdrop of the S's direct contacts constitute "minimum contacts" such that it does not offend traditional notions of fair play and substantial justice.

Levin v. Harned
FACTS: The Levins live in Boston. They sought to furnish both with authentic and valuable antiques. They hired an interior designer to recommend and purchase the antiques. The designer, Harned, met with the couple and presented photographs and descriptions of antiques from various suppliers. On October 31, 2000, the Levins wired Harned $4.9 million. Dalva Brothers is an antique dealer based out of New York. Dalva had previously done business with the Levins and knew they lived in Massachusetts, were aware that the Levins were purchasers of the items sold and that all the items were shipped to Massachusetts. Dalva dealt with Harned. Harned made the shipping arrangements for the Levins. Dalva gave Harned pictures that it knew he would give to the Levins. Dalva is not "authorized" to do business in Massachusetts, has no employees, agents, property, advertising or soliciting in Massachusetts. Foster-Gwin is pretty much the same except that it has made two small sales to Massachusetts residents. Foster-Gwin is based out of San Francisco. Hardy is also based out of San Francisco, and is also in the same boat.
REASONING: The Specific Jurisdiction is established because the company contracted to supply things in the commonwealth, caused tortious injury by an act or omission outside the commonwealth. The cause of action is related to the defendant's contacts with the forum, and the contacts constitute purposeful availment. The distributors chose to do business with the forum resident and received a benefit from that business. The defendant committed an intentional tort, the forum was the focal point of the harm, the forum was the focal point in that it was expressly aimed at the forum. The defendant's conduct and connection with the forum state are such that he should reasonably anticipate being haled into court there. The defendants knew that Harned would transmit their fraudulent information to the Levins in Boston and that the Levins would make a purchasing decision based on this information. The defendants released the goods to a Massachusetts shipper knowing the antiques were being shipped to Massachusetts.

Ø CYBERSPACE CASES

Zippo Manufacturing Co. v. Zippo Dot Com
FACTS: Manufacturing is a PA corporation with its p/p/b in Pennsylvania. Manufacturing makes the well known "Zippo Lighters." Dot Com is a California corporation with its p/p/b in California. Dot Com operates an internet Web Site and an Internet news service and has obtained exclusive right to use the domain names zippo.com, zippo.net and zipponews.com. Dot Com's Web site contains information about the company, advertisements and an application for Internet news service. The news service itself consists of a free news, Original and Super. A customer who wants to subscribe to the Original or Super fills out an on-line application that asks for a variety of information including the person's name and address. Payment is made by credit card over the Internet or telephone. Dot Com maintains no offices, employees or agents in Pennsylvania. Dot Com's advertising for its service to PA residents involves posting information about its service on its web page which is accessible to PA residents via the Internet. Two percent (3,000) of those subscribers are PA residents. Dot Com has entered into agreements with seven Internet access providers in PA to permit their subscribers access to Dot Com's news service. Zippo is filing suit for trademark infringement.
REASONING: Pennsylvania's long arm jurisdiction statute authorizes jurisdiction over non-resident defendants upon contracting to supply services or things in the commonwealth. Specific jurisdiction applies here based on the three pronged test. The defendant must have minimum contacts, the claim asserted against the defendant must arise out of those contacts (relatedness) and the exercise of jurisdiction must be reasonable(Gestalt). Dot Com has contracted with 3,000 individuals and the intended object of the transactions was the downloading of messages. Dot Com cannot maintain that these contracts are "fortuitous" or "coincidental." If a corporation determines that the risk of being subject to personal jurisdiction in a particular forum is too great, it can choose to sever its connection to the state. Trademark infringement is a tort-like injury and a substantial amount of the injury from the alleged infringement was likely to occur in Indiana. The cause of action therefore arises out of Dot Com's forum-related activities.
NOTE: This case established the "sliding scale" test which requires that both the party in question "clearly does business over the internet" and that the internet business contacts with the forum state be substantial or continuous or systematic.

Minimum Contacts and the Internet:
Look at the nature and quality of commercial activity Defendant conducts over the internet.
Uses a sliding scale approach: ESTABLISHED BY ZIPPO
· Active - Defendant clearly does business over the internet
· Passive - Does no more than make information available. No sales or anything to do business.
· Middle Ground - Sliding Scale - What has the requirement?
Look at the level of interactivity and commercial nature of the exchange. People paying money for the services?

Internet Test
a. D must direct electronic activity into the state
b. With intent of engaging in business or other interactions within the state, and
c. That activity gives rise to the cause of action by person within the state

Gator.com Corp. v. L.L. Bean. Inc.
FACTS: L.L. Bean is a Maine corporation with p/p/b in that state. A large percentage of LL Bean's sales come from mail-order and internet business. LL Bean is not authorized to do business in California, has no agent for service of process in California, and is not required to pay taxes in California. In 2000, LL Bean mailed a substantial number of catalogs and packages to CA residents and targeted substantial numbers of CA residents for direct email solicitation. . Plaintiff is a Delaware corporation with its p/p/b in California. Gator develops and distributes software to consumers who purchase goods and services over the internet. The software is a "digital wallet" that remembers passwords and credit card numbers to various websites. When it recognizes certain URLs that have been preselected by Gator, the program will create a pop up window offering a coupon for a competitor. For users visiting LL Bean's website, an Eddie Bauer pop up appears. This is a cease-and desist order.
REASONING: For general jurisdiction, there must be substantial or continuous or systematic contacts. There are these contacts based on LL Bean's extensive marketing and sales in CA, its extensive contacts with California vendors and the fact that its website is deliberately structured to operate as a sophisticated virtual store in California. The sales are also consistent with the "Sliding Scale" test that both the party in question clearly does business over the internet and that the internet business contacts with the forum state be substantial or continuous and systematic. Under the sliding-scale analysis, LL Bean's contacts with CA are sufficient to confer general jurisdiction. LL Bean's website is highly interactive and very extensive. LL Bean clearly does business over the internet. There is substantial evidence that LL Bean has purposefully interjected itself into the CA market. The burden on Gator if it were forced to proceed in Maine would be at least equal to if not more severe, than the burden faced by LL Bean.

Young v. New Haven Advocate
FACTS: The State of Connecticut was faced with substantial overcrowding in its maximum security prisons. To alleviate the problem, Connecticut contracted with Virginia to house Connecticut prisoners in Virginia's correctional facilities. Newspapers began reporting on the situation, and some wrote stories discussing the allegedly harsh conditions at the Virginia prison. The warden was not mentioned in any of the columns, but he was mentioned by a state senator who expressed concern about the presence of Confederate Civil War memorabilia in his office. On May 12, 2000, Warden Young sued the two newspapers, editors, reporters in a libel diversity action claiming that the newspapers' articles imply that he is a racist who encourages abuse of inmates by the guards. The Advocate, one of the newspapers being sued, is a free newspaper published once a week in New Haven, Connecticut. It is distributed in New Haven and the surrounding area and some content is published on the internet. There are a small number of subscribers, but none are in Virginia. The Courant is published daily in Hartford, distributed in and around Hartford, and some content is also published on the internet. The Courant has 8 mail subscribers in Virginia. Neither newspaper solicits subscriptions from VA residents, no one traveled to VA to work on the stories, there were a few telephone calls to VA to gather some information for the articles. Warden Young points out that the newspapers posted the allegedly defamatory articles on websites accessible to VA residents. The website printouts submitted do not contain any information targeting Virginia.
REASONING: Warden Young argues for specific jurisdiction which requires purposeful availment, a C/A arising out of forum related activities and the reasonability. The fact that the newspapers' websites could be accessed anywhere, including Virginia, does not by itself demonstrate that the newspapers were intentionally directing their website content to a Virginia audience. The overall content of both websites is decidedly local, and neither newspaper's website contains advertisements aimed at a Virginia audience. These newspapers maintain their websites to serve local readers in CT, to expand the reach of their papers within their local markets and to provide their local markets with a place for classified ads. Connecticut, not Virginia was the focal point of the articles. Furthermore, the newspapers could not reasonably anticipate being haled into a Virginia court. The Newspapers do not have sufficient Internet Contracts with Virginia to permit the district court to exercise specific jurisdiction over them.

Revell v. Lidov
FACTS: Lidov wrote a lengthy article on the subject of a terrorist bombing. The article singles out Revell, then Associate Deputy Director of the FBI, for severe criticism, accusing him of complicity in the conspiracy and cover-up. The article further charges that Revell, knowing about the imminent terrorist attack, made certain his son took a different flight. The article was posted on the Columbia University bulletin board through the CU School of Journalism. Lidov has never been a student or faulty member of Columbia, and he did not know at the time that Revell resided in Texas. Revell claims damage to his professional reputation in Texas and emotional distress arising out of the alleged defamation of the defendants, and sought several million dollars in damages.
REASONING: The cited contacts of Columbia with Texas are not in any way substantial. Columbia's boards are found to be interactive, which makes the site more than just passive. This must be evaluated with respect then to the "effects" test set out in Calder. The article written by Lidov about Revell contains no reference to Texas, nor does it refer to the Texas activities of Revell, and it was not directed at Texas readers. Texas was not the focal point. Lidov's article deals exclusively with his actions with the FBI, there is no reference to Texas. The post to the bulletin board here was directed at the entire world, but not specifically at Texas. Due process requires that the defendant's conduct and connection with the forum state are such that he should reasonably anticipate being haled in there. Lidov states that he never even knew while posting the article that Revell lived in Texas which means he never knew where the brunt of the harm would be felt. A more direct aim is required than there is here.

Ø JURISDICTION OVER PROPERTY

Shaffer v. Heitner
FACTS: Heitner is a nonresident of Delaware and owns one share of stock in the Greyhound Corp, a Delaware Corp. with p/p/b in Phoenix, Arizona. Heitner alleged that the individual defendants had violated their duties to Greyhound by causing it and its subsidiary to engage in actions that resulted in the corporations being held liable for substantial damages in a private antitrust suit and a large fine in a criminal contempt action. The activities which led to these penalties took place in Oregon. Heitner filed a motion for an order of sequestration of the Delaware property of individual defendants. The stock was considered to be in Delaware, and so subject to seizure.
REASONING: The Delaware courts rejected appellants' jurisdictional challenge by noting that this suit was brought as a quasi in rem proceeding. Since quasi in rem jurisdiction is traditionally based on attachment or seizure of property present in the jurisdiction, not on contacts between the defendant and the State, the courts considered appellants' lack of contacts with Delaware to be unimportant. The case for applying to jurisdiction in rem is the same test of fair play and substantial justice is simple and straightforward. In order to justify an exercise of jurisdiction in rem, the basis must be sufficient to justify exercising jurisdiction consistent with the minimum-contacts standard. In this case there were no real minimum contacts at the expense of fairplay. Also, the appellants' holdings do not provide contacts with Delaware sufficient to support the jurisdiction of that state's courts over appellants. Delaware law bases jurisdiction on the presence of their property in the State. It does not demonstrate that appellants have purposefully availed themselves of the privilege of conducting activities within the forum state. Appellants had nothing to do with the forum state nor did they have a reasonable expectation of being haled into court in the state.

Burnham v. Superior Court of California
FACTS: Burnham married Francie Burnham in 1976 in West Virginia and moved to New Jersey the next year where they had two children. In July, 1987 the couple decided to separate and it was agreed upon that Francie would have custody of the children and live in California and the two would divorce due to irreconcilable differences. In October, Burnham filed for divorce and listed the reason as desertion. After learning of this, Francie filed for divorce in California in January 1988. In late January, Burnham visited southern California on business, and visited his child as well. Upon returning the child to Francie's home, he was served with a California court summons and a copy of Francie's petition. Burnham then returned to New Jersey.
REASONING: A valid jurisdictional predicate for in personam jurisdiction is that the defendant was present in the forum state and was personally served with process. Burnham's argument misunderstands many of the cases. Nothing says that a defendant's presence in the forum is not only unnecessary to validate jurisdiction, but it is no longer sufficient. Jurisdiction based on physical presence alone constitutes due process because it is one of the continuing traditions of our legal system that define the due process standard of "traditional notions of fair play and substantial justice."



Service of Process

Ø RULE 4

To satisfy Due Process requirements, notice to the defendant must be of a quality that is reasonably likely, in all the circumstances of the case to apprise the defendant of the pending action and afford an opportunity to make a defense

A) Summons needs to be signed by the clerk, bear the seal of the court, id parties, if the person doesn’t show, they lose
B) Upon filing the complaint, the plaintiff may present a summons to the clerk for signature and seal. A summons or a copy of the summons if addressed to multiple defendants shall be issued for each defendant to be served.
C) Service
1) A summons shall be served together with a copy of the complaint. The plaintiff is responsible for service and within a certain time.
2) Service may be effected by anyone not in a party and 18 or older. At the request of P, a sheriff or US Marshal, or deputy or other person specially appointed by the court may do so.
D) Waiver of Service
1) A defendant who waives service does not waive any objection to the venue or to the jurisdiction of the court.
2) An individual/corporation has duty to avoid unnecessary costs of summons. He may notify defendant and request that the defendant waive service.
a. The notice and request shall be in writing and addressed directly to the defendant or agent authorized to receive service of process
b. Shall be dispatched through first-class mail
c. Shall include a copy of the complaint and ID the court it was filed in
d. Inform the defendant of consequences of failure to comply with request
e. Date on which request is sent
f. Allow defendant a reasonable time to return the waiver, at least 30 days, or 60 if addressed outside any judicial district of the US
g. Shall include an extra copy of the notice and request as well as a prepaid means of compliance in writing.
Failure to comply will result in the costs imposed for effecting service unless good cause for failure can be shown
3) A defendant that returns a waiver so requested is not required to serve an answer to the complaint until 60 days after the date on which the request for waiver of service was sent or 90 if outside a US District
4) If plaintiff files a waiver of service it is the same except as in paragraph 3
5) The costs imposed on a defendant for failure to comply with a request to waive service of a summons shall include costs in E, F, or H.
E) Unless otherwise provided, service upon an individual other than an infant or incompetent person may be effected in any judicial district
1) Following the state law or
2) By delivering a copy of the summons/complaint to the individual personally or leaving copies at the house or usual place of abode with person of suitable age and discretion residing therein or to an agent authorized to receive it.
H) Service on a corporation is subject to suit under a common name and from which a waiver has not been obtained an filed, shall be effected
1) In a judicial district of the US in the manner of state law, or delivering to an offer, or agent authorized to receive it.
2) In a place not within any US district in any manner prescribed except personal delivery which requires someone special
K) Territorial limits of effective service
1) Service of a summons is effective to establish jurisdiction over a defendant
A. Who could be subjected to the jurisdiction of general jurisdiction of a state or
B. Who is a party under Rule 14 or 19 and is served at a place within a judicial district of the US and not more than 100 miles from the place where summons was issued or
C. Subject to federal interpleader jurisdiction
D. Statute authorization
2) If the exercise is consistent with the Constitution, servicing a summons or filing a waiver is effective for federal law claims to establish personal jurisdiction over the person of any defendant who is not subject to the jurisdiction of the courts of any state.
M) Time limit – If service of the summons and complaint is not made upon defendant within 120 days after filing of the complaint the court on motion or its own initiative shall dismiss the motion. Extensions for good reason are allowed.

Where you can serve someone:
1) Personally delivering to the defendant
2) Leaving a copy of summons/complaint at the dwelling house with a person of suitable age and discretion
3) Leaving the copy with an agent
4) Using the provisions set forth in the court the suit is filed in. (Mass Fed court = Mass rules)
5) Using the provisions set forth in the state where service is effected (Person is in Maine = Maine rules even if suing in a Mass Fed Court)

Waiver of Process
1) Duty to avoid unnecessary costs of summons
2) Costs if there is no good cause why you didn’t waive
3) 60 day response if waived as compared to 20 days if not waived

Federal Long Arm Statute
1) If state court could have jurisdiction then so could federal court. (4K)
2) If you live within 100 miles of the courthouse (regardless of state lines) 4K1b
3) Interpleader jurisdiction 4K1c
4) Jurisdiction in the US as a whole, but not the state
5) Must be made within 120 days of filing of complaint 4M
a. In some states the statute of limitations requires that complaint/summons be SERVED while in others it only needs to be FILED.


Under Pennoyer, if service is not made properly, personal jurisdiction was not obtained. Both must be analyzed separately




Subject Matter Jurisdiction - Decides what types of cases courts can hear

Three reasons why a Federal Court can hear a case:
1) The case raises a federal question
2) The parties are diverse in their domiciles
3) Supplemental Jurisdiction

While defects in PJ may be waived if no timely objection, defects in SMJ never waived. You cannot waive the SMJ. It goes to the power of the court, the structure of the constitution. If the federal court doesn't have SMJ, that is not a defense that can ever be waived. A judge or the parties may bring up the SMJ contention.

U.S. Art III, Section 2 provides for kinds of "cases or controversies" to which judicial power shall extend.
One provides that judicial power shall extend to all cases arising under the Constitution, laws or treaties of the U.S.



Ø FEDERAL QUESTION

28 USC §1331 – Federal Question
1) Well Pleaded Complaint Rule – The federal question must appear on a fair reading of the well pleaded complaint and not in the defense. It must be raised by the Plaintiff! This is also known as a Mottley Test
2) Congressionally Chartered Corporations – If created by a congressional enactment, they may sue or be sued in federal court regardless of what the claim is.
3) Constitutional Torts – Supreme Court has decided some torts can be decided in federal court
4) State Claim Dependant on Federal Law
5) Jurisdiction Decided When? - Jurisdiction is generally determined at the time the complaint was filed, events occurring subsequent to the filing normally have no bearing on the court’s jurisdictional decision.
6) Minimum Amount Required - No amount is required


Louisville & Nashville R.R. v. Mottley
FACTS: The Mottleys, in 1871, while passengers on the railroad were injured by the railroad's negligence and released their respective claims for damages in consideration of the agreement for transportation during their lives. A contract was performed by the railroad up until 1907, when the railroad declined to renew the passes. The refusal to comply was based on the act of Congress in June of 1906. This forbids the giving of free passes or free transportation.
REASONING: The main question of law were raised here, whether the act of Congress makes it unlawful to perform the contract which was accepted by both parties in satisfaction and made previous to the passing of the act can still be enforced. Therefore, the issue is over a breach of contract, and not about the act. There was no diversity of citizenship. The Federal Question requirement of Federal Jurisdiction must be raised in the complaint, and not in the defense which was done in this case. Therefore, there is no federal jurisdiction. **The main issue here was that the Railroad was sued for performance, and they tried to remove to federal court based on Federal Question, however, the court found that the federal question must be raised in the complaint ONLY, and not as a defense (Well Pleaded Complaint Rule).

Nashoba Communications v. Town of Danvers
FACTS: On December 23, 1985, the town of Danvers granted Nashoba a license to construct and operate a cable television system. It guaranteed its proposed installation as well as economy and super basic service rates, which would not rise for two years. Any future changes in the initial schedule of rates and charges would be in conformance with the Cable Act. On June 14, 1988, Nashoba sent a letter to cable subscribers and Board of Selectmen informing them that they planned to eliminate economy basic and also to raise prices for the super basic cable service rate. The Danvers' Town Council replied with a letter calling this increase premature. Nashoba then filed suit against the rate freeze provision of the license.
REASONING: The cable operator argues that they want enforcement of the Cable Act. This is the defense brought up by them in regards to Danvers requesting fulfillment of a contract. Since this is a defense, it is not raising the federal question and therefore, not in the jurisdiction of a federal court.



Ø DIVERSITY OF PARTIES

28 USC §1332 – Diversity of Parties - Permits a federal district court to hear state causes of action if two requirements are fulfilled
1) The plaintiff must be a citizen of a state other than that in which the defendant is a citizen. Strawbridge v. Curtiss
a. The parties must be diverse at the time the action is filed.
b. All plaintiffs must have diversity from all defendants
2) The amount in controversy must exceed $75,000 (No interest or costs allowed)


Citizenship requirements
1) When the plaintiff and defendant are from different US States
2) When the parties on one side are from a US state and the other is from a foreign country
3) When a foreign state is a plaintiff suing citizens of American States



ü DOMICLE

Domicile – A person is domicile where he is physically present with an intent to remain indefinitely. This includes aliens.
Exemptions: - Domestic Relations Ankenbrandt 1992 (Divorce, Alimony, Child Custody) and Probate

Corporate Citizenship 1332 (C) - A corporation is a citizen for diversity purposes where it is incorporated and the place in which it has its principal place of business. If incorporated in more than one state, then it is a citizen of every state of incorporation.
Principal Place of Business – The location of the corporation’s headquarters, or the place where the bulk of the corporation’s assets may be found. “Total Activity Test”

****IMPORTANT NOTE: If the corporation has many jurisdictions of citizenship, the plaintiff must be diverse from all of them

Unincorporated Associations – Citizens of each state in which one of their members is a citizen (Major League Baseball)

Two Foreign Parties – There is no rule allowing someone from Britain to sue someone from Brazil in a US Federal Court. An American party must be attached.

US Citizens Domiciled in Foreign Countries – Cannot be sued in Federal Court under Diversity purposes.

Fraudulent Joinder – When a plaintiff who has sued in state court joins a nondiverse defendant against whom the plaintiff obviously has no cause of action. The purpose is to prevent removal to federal court by destroying diversity. If the court identifies this, they may allow the federal court to hear the case.

Garcia Perez v. Santaella
FACTS: On May 6, 1996, the Garcias had premature quadruplets only one of which survived. The babies were born in Puerto Rico, but following the death of three of the children moved to Miami. On May 5, 1997, the medical malpractice claim was filed. The defendant claimed that there was no subject matter jurisdiction. The Garcias are registered to vote in Florida, have drivers' licenses in Florida, sold their car in Puerto Rico and purchased two cars in Florida and both passed the Florida Bar Exam. They opened a Miami bank account, which is their primary account for the family's expenses. They did at some point someday hope to return to Puerto Rico.
REASONING: A "floating intention" to return to a former domicile at some unspecified future date does not prevent a party from acquiring a new domicile. This is exactly what Carlos' intention was. In this case, it was shown that Carlos and Gisela created substantial personal, professional and civic ties to Florida that significantly outweighed their residual ties with Puerto Rico. Florida was their personal and financial base. It is clear that Carlos had the settled intent to practice law in Florida with reasonable expectation of an association in Florida with his firm. Where you are domiciled at the time of filing is what governs domicile.


American Fiber & Finishing, Inc. v. Tyco Healthcare Group
FACTS: On February 16, 2001, plaintiff sued the defendant in Massachusetts. In its complaint, AF & F claimed that Tyco International was liable as a successor in interest to Kendall Company for response costs incurred in connection with the decontamination of an industrial site in Massachusetts. Federal jurisdiction attached based on diversity of citizenship. AF & F was a Delaware corporation with p/p/b in North Carolina and Tyco International is a Nevada corporation with p/p/b in New Hampshire. The plaintiff then amended its complaint by dropping Tyco International and inserting Tyco Healthcare. The reason for the switch is that Tyco Healthcare was a successor company of the Kendall Company which AF & F really wanted to go after but could no longer due to the company going out of business probably. Tyco Healthcare is a Delaware limited partnership and is incorporated in that state. The two companies held the same citizenship.
REASONING: At the time of arrival on the scene, Tyco Healthcare destroyed the requisite diversity and thus, eliminated any chance of federal matter jurisdiction. AF & F could not originally have brought suit in federal court against Tyco Healthcare, since citizens of Delaware would have been perched on both sides of the litigation. When it amended its complaint, complete diversity was destroyed. The transfer occurred prior to the beginning of litigation. Although one might find this an excuse, AF & F might have won the first round, and then on appeal lost due to subject matter jurisdiction.

ü $75,000 MINIMUM


Legal Certainty ($75,000) Test (St. Paul Mercury Test) - Only when it is certain that the plaintiff cannot claim more than $75,000 will the court not allow diversity. Plaintiff’s burden of proof. For defendant to remove he must show that “it is not a legal certainty that the plaintiff cannot recover more than $75,000”

Aggregation of Claims – In order to meet the $75,000 minimum, a plaintiff may add the value of multiple claims against a defendant to satisfy the minimum, no matter how unconnected the claims may be. If each claim is an alternative theory to recovery under one cause of action, then it may be added only once. Two or more people may aggregate their claims if they are truly joint, common and united. This is usually for property or joint trusts.

Aggregation Rules
Single Plaintiff may aggregate claims versus single defendant
Multiple plaintiffs can't aggregate separate and distinct claims
Snyder v. Harris (1969)
Class action suit, Supreme Court opinion
You may have 3,000,000 members of a class, and each may have a $10,000 in claims, but you cannot aggregate those individual claims to reach the necessary amount for diversity of action under Subject Matter Jurisdiction
Zahn v. International Paper Co. (1973)
Supreme Court decision that said that "even if one member of that class has the requisite amount in controversy, the other class members cannot ride in on the coattails of that member. Their claims will not come in under a concept of supplemental jurisdiction.
If plaintiff has a $40,000 property claim and a $45,000 personal injury claim in the same injury in the same accident, then the claims can be aggregated.
If the claims are unrelated, $40,000 for property, and $45,000 personal injury arising out of a separate occurrence against same defendant … they can STILL be aggregated. You can put in any and all claims against one person and aggregate them. As long as it is single plaintiff v. single defendant
Two plaintiffs v. defendant and each plaintiff seeks $40,000, there is no diversity of action because you cannot aggregate them.
Two plaintiffs JOINTLY seeking $80,000 through maybe a property or a trust fund. In this situation, the jurisdiction is OK as long as it is "common and undivided"
Plaintiff v. 2 Defendants - P seeks $40,000 from each, no jurisdiction here


Stewart v. Tupperware Corp.
FACTS: Plaintiffs were a recently married couple who while driving down a street during their honeymoon were struck by a car driving the wrong way down a street and running a red light. The wife suffered whiplash, chest trauma, cuts, bruises, continuous pains prohibiting her from using 3% of her body and breast feeding. The husband had whiplash and lost 7% of his mobility.
REASONING: Plaintiffs brought claims for past and future mental and emotional pain and suffering. Loss of enjoyment from life, impairment and future medial costs. The determination of the value of the matter in controversy for purposes of federal jurisdiction is a federal question to be decided under federal standards. Using the Puerto Rico Supreme Court is like comparing apples to oranges. Under the federal standard, the 75,000 is met.

In re Ford Motor Co./Citibank
FACTS: In early 1993, Ford and Citibank issued a cobranded credit card offering cardholders the opportunity to save on the purchase or lease of a new Ford, up to $3500 through 5 years. On December 31, 1997, less than 5 years after the program's inception, Ford and Citibank terminated the rebate accrual feature of the credit card. Six state actions were filed and Ford and Citibank tried to remove the cases to the Federal Court Level.
REASONING: The defendant tries to do two tests, combining the parties as one and also claiming that their own damages as a whole to each corporation will be at least $75,000. The court reasons that each plaintiff's claim must be held separate from each other plaintiff's claim from both the plaintiff's and the defendant's standpoint. Each must be separately evaluated. There were no "common and undivided interests." The consolidated plaintiffs in this case did not unite. Also, the fixed administrative costs to the defendant cannot satisfy the damages requirement. Once again there are no common and undivided interests.


Ø SUPPLEMENTAL JURISDICTION

Pendant Jurisdiction – Plaintiff asserted a jurisdictionally proper claim against a nondiverse party and added on a related state claim. So long as the plaintiff asserts a proper claim based on federal law, diversity, or some other federal ground, the federal court has the power to hear other claims arising out of “common nucleus of operative facts.” United Mine Workers v. Gibbs.

3 Part Test
1) Article III Power Question - (Common Nucleus)(Gibbs)
2) Statutory Question – (Is there a statute requiring/forbidding this?)(Aldinger)
3) Discretion Question – (Does it make sense to hear this case)(Gibbs)


Ancillary Jurisdiction – Third party claims that would normally be required to be heard in state court. Since they are “logically related” to the original claim and both a part of the same “constitutional case.” Example, A sues B in a Federal Question, both from same state. B counterclaims on a state claim. This is allowed if it is based on the same event or claim. Ancillary parties are third parties not joined by the plaintiff. Owen v. Kroger

May be dismissed if:
1) State claims too complex
2) State claims predominate
3) Federal Question Dismissed
4) Some other crazy reason in an exceptional circumstance

28 USC §1367 - Codification of the Goodness
a. (a): In any civil action in which district courts have original jurisdiction, they shall have supplemental jurisdiction over all other claims so related to jurisdictional conferring claims that they form part of the same case or controversy under Art. III.
b. (b): Places limitations on supplemental jurisdiction when original jurisdiction is based on diversity
i. If you’re in a federal court and your basis for jurisdiction is just diversity, many claims supplement jurisdiction will not apply
c. (c): Codifies factors to be considered by court in exercising discretion to hear/ not hear supplemental claims
d. Supplemental jurisdiction May be declined if:
i. Novel/ complex issue of state law
ii. State claims predominate
iii. Federal claims dismissed
iv. In exceptional circumstances, other compelling reasons
e. 3 – Part Analysis:
i. Constitutional Power (Gibbs)
ii. Statutory Grant of Jurisdiction (Mostly §1367)
iii. Discretion Analysis


Ø REMOVAL JURISDICTION

Removal allows a defendant to move a case from the state trial court to a federal district court.
Removal: 28 USC §1441
a. §1441 (a): Where civil action filed in state court is one that could have been brought originally in federal court, D(s) may remove to district court embracing place where action is pending. Can be removed without regard to citizenship of D
b. §1441 (b): If federal question jurisdiction, D can remove without regard to citizenship or residence of D. If diversity jurisdiction, not removable if any D is a citizen of state where action was filed, even though it’s a case that could have been brought originally in federal court. Local citizen D doesn’t need to invoke federal jurisdiction b/c chances of prejudice operating will be minimal
28 USC §1446(b) = D has 30 days to file notice of removal, no case can be removed after a year in the state court system
28 USC § 1367(d): If a case is removed to federal court, the statute of limitations will be tolled while the case is pending in the federal court, statute of limitations on state law claims will be tolled
28 USC §1447(e): If at any time before final judgment it appears that district court lacks subject matter jurisdiction then the case shall be remanded to state court
28 USC §1292(b): general rule is that you can only make appeal after final judgment. If court decides it’s an important question of law of which there is disagreement, court lets appeals court hear the case immediately rather than wait until the end of the suit because it would be more difficult. Court certifies order for immediate appeal under this statute


Requirements
1) The claim falls within proper subject matter jurisdiction.
2) The case must have been filed in state court at the time the removal petition was filed.
3) Defendants on crossclaims, counterclaims, or third-party impleaders have not right to remove cases from the state courts. (1441a)
4) There must be unanimous consent among all defendants charged at the time.
5) Cases removed from the state court are removed to the federal court of the district that includes the location in which the state court sits
6) If the federal court has original jurisdiction over the claim – Federal Question (1441b)
7) Where the parties meet the requirements of diversity jurisdiction except for any defendant being a citizen where the case was filed (1441b)
8) There is no pre-arranged agreement to stay in state court.
9) If there is removal in a case against a foreign country, it may be removed regardless of federal question of the $75,000 rule, and it must be tried without a jury.

Joinder of “Non Removable” Claims
If there is one claim that is removable, all claims will be removable like in supplemental jurisdiction. This includes crossclaims, counterclaims, and claims against third parties.

However, if there is a nondiverse state claim, it must stay in state court.

If the claims from the state are predominant, then the case will be remanded.

Rose v. Giamatti
FACTS: Plaintiff was the manager of the Cincinnati Reds and the defendant was the Commissioner of Major League Baseball. The Commissioner was going to begin an investigation into allegations of Rose's gambling on baseball and Rose set out to file injunctions stopping this. The matter was taken to a district court of Columbus before a motion was filed to remove the matter to the US District Court for Southern Ohio. The plaintiff appeals seeking to deny removal based on no diversity of parties. In his action he has listed Giamatti, the Cincinnati Reds, a citizen of Ohio and Major League Baseball, a citizen of Ohio among other states, as defendants. Giamatti, a citizen of New York seeks to have this kept in the US court based on the Reds and MLB being nominal parties.
REASONING: The citizenship of MLB is strange because it is an unincorporated association created by agreement. The joinder as a defendant would not defeat the required diversity of citizenship. An unincorporated association has no citizenship of its own, but is a citizen of every state in which each of its constituent members is a citizen. For diversity purposes, this includes Ohio. Therefore, it must be looked into whether or not they are a nominal defendant or a real defendant. The case is primarily against Giamatti based on his name being the only real name listed in the complaint. The real question is with the Reds and MLB. Rose alleges no wrongful conduct on the part of the Reds. There is no real controversy between the parties. The Reds have been fraudulently joined. The same can be said for MLB. Rose also claims that Giamatti through contesting the injunctions and whatnot waived his right to remove the trial. This is also wrong because there needs to be a complete trial and an end to the trial before one can decide that the right has been waived. There was no litigation on the merits of the dispute. There was no action to affirmatively embrace jurisdiction.

Lewis v. Caterpillar, Inc.
FACTS: The plaintiff was a Kentucky citizen and the defendants were DE/IL citizens and a KY citizen (Whayne). Liberty Mutual (MA) then joins in as a plaintiff, so it is 2 on 2. Lewis settles with Whayne and Caterpillar tries to remove to Fed Court (with one day left on the clock.)
REASONING: Whayne had not been dismissed as a defendant because the other plaintiff (Lib Mut) had a claim against them. Furthermore, the case was tried in a KY state court and since Whayne is a KY citizen, the case cannot be tried there. However, if the defect is cured before judgment is rendered, there is no reason to remand the case back to state court.

Groupo Dataflux v. Atlas Global Group, L.P.
FACTS: Atlas is suing Dataflux for a breach of contract in So. Texas. 2 of 5 partners were Mexican citizens. So Atlas is a citizen of Mexico, Delaware and Texas. 2 non-diverse partners removed from partnership before trial, so at the time the case went to trial, you had complete diversity.
REASONING: No party was dismissed here, there was simply a change of citizenship. There was no diversity at time of filing.



Venue

If it is a removal, there is only one possible venue, and that is the district court which embraces the original state court.

28 USC §1391 (a)
Where diversity = sole basis for subject matter jurisdiction, venue is ok
If your case is in federal court under diversity jurisdiction you have 3 choices:
1. District where any defendant resides if all defendants reside in the same state
2. In a district where a substantial part of events or omissions giving rise to claim occurred or where substantial part of property subject of suit is located or…..
3. In a district in which any defendant is subject to personal jurisdiction at time action is commenced, if no district in which action may otherwise be brought (THIS ONE IS RARELY USED)
1391 (b)
Where there is a federal question
If your case is in federal court under a federal question, venue is ok when:
1. In a district where any defendant resides if all Ds reside in the same state
2. In a district where a substantial part of events or omissions giving rise to claim occurred or where substantial part of property subject of suit is located or…..
3. In district where any D may be found, if there is no district in which the action may be otherwise be brought. (INTENTIONALLY DOES NOT SAY PERSONAL JURISDICTION)
Note for 1391 (a) and (b)
Individual Defendant "resides" in district where domicile located


Ø Rules of Decision Act
28 USC §1652 Rules of Decision Act = the laws of the several states, except where the Constitution, US Treaties or Acts of Congress otherwise require/ provide, shall serve as rules of decision in federal courts in cases where they apply

Erie R.R. v. Tompkins
FACTS: The P was walking along a footpath running parallel to the Erie RR tracks when the door protruding from a passing train struck him and his right arm was severed. Under PA law a traveler like P on a parallel path was regarded as a trespasser to whom the railroad merely owes a duty to avoid wanton negligence. The majority rule in most states however was that a railroad owes a duty of ordinary care to a traveler on a parallel footpath. His lawyers tried to avoid the harsh PA rule by suing the NY based railroad in federal court.
REASONING: Court held that congress has no power to declare substantive rules of common law applicable in a state whether they are local or general. Except in matter governed by the federal constitution or by acts of congress, the law to be applied in any case is the law of the state. State substantive law and federal procedural law apply where the federal court is enforcing a state created right.

Guaranty Trust v. York
FACTS: York brought a diversity suit alleging that Guaranty had breached its fiduciary duties. York’s complaint involved allegations of fraud and misrepresentation, relief for which was governed by equitable principles. Issue was when no recovery could be had in a state court b/c the action is barred by the statute of limitations, a federal court in equity could take cognizance of the suit b/c there is diversity of citizenship between the parties.
REASONING: Court ruled that since a federal court adjudicating a state created right solely b/c of the diversity of citizenship of the parties is for that purpose in effect only another court of the state, it cannot afford recovery if the right to recover is made unavailable by the state nor can it substantially affect the enforcement of the right as given by the state. If outcome would be different in the federal court than it would be in the state court, then that state law must be treated as substantive for Erie purposes.

Byrd v. Blue Ridge Co.
FACTS: Petitioner sues respondent for damages for injuries allegedly caused by the respondent’s negligence. One of the respondent’s affirmative defenses was that under the South Carolina’s workmen’s comp act, the petitioner had the statute of a statutory employee of the respondent and was therefore barred from suing the respondent at law and obliged to accept statutory comp benefits instead.
REASONING: Court held that the interest in uniformity of outcome is not the only interest, must also consider federal interest and state statutes which would interfere with the appropriate performance of that function are not biding upon the federal court under either the conformity act or the rules of decision act. Draw back from the strict outcome approach to consider any strong countervailing federal interest. Federal interest in running its own court outweighs against the interest of uniformity of outcome.



REASONS FOR RDA
1. In absence of controlling federal law, the federal court should apply state substantive law when enforcing a state created right (Erie) (no statute, federal rule or constitution on point)
2. Even if the state law is arguably procedural (e.g. statute of limitations) still apply the state law in the interest of achieving uniformity of outcome in enforcement of state created rights (York)
3. Especially where ignoring or displacing state law would affect the outcome so as to:
Either encourage forum shopping or
Result in unfairness in administration of laws (Hanna Part 1)
4. Unless there is a strong countervailing federal interest (Byrd)


Ø Rules Enabling Act

28 USC §2072 Rules Enabling Act = delegation of authority to Supreme Court the power to make rules governing practice and procedure in the lower federal courts, district and court of appeals. Rules should not abridge, enlarge or modify any substantive right.

Notes:
1) If there is a Federal Rule of Civil Procedure on point, question whether it complies with REA
2) Strong presumption that when the federal rule really regulates procedure, and it does not abridge, enlarge or modify any substantive rights then it should be applied

Hanna v. Plumer
FACTS: Petitioner a citizen from OH filed her complaint in the district court of MA claiming damages from an automobile accident in SC allegedly caused by the negligence of Plumer a MA citizen. Issue as to whether in a civil action where the jurisdiction of the US district court is based upon diversity of citizenship between the parties, service of process shall be made in the manner prescribed by state law or that set forth in rule 4(d)(1) of FRCP.
REASONING: Court ruled that when a case has an applicable federal rule in direct collision with the law of the relevant state, the federal rule is valid and controls. The federal rule on point really regulates procedure and does not abridge, enlarge or modify any substantive right.

Walker v. Armco
FACTS: Negligence suit in diversity with a 2 year statute of limitations. The complaint was filed in August of 1977 but the defendant was not served with process until December of that year. Issue as to whether in diversity suit, does FRCP 3 or state law govern when action commenced for purposes of tolling the statute of limitations.
REASONING: Court held that FRCP 3 simply was not meant to govern tolling of state statute of limitations b/c the federal rule did not apply b/c it wasn’t meant to govern tolling. Therefore, once there is no federal rule on point, determine that if don’t apply state law on tolling it would impact the outcome of litigation, therefore state law is treated as substantive and must apply in federal court.


Ø Transfer of Venue

Forum non-conveniens
1) Common law doctrine – judge made
2) Two conditions must be satisfied before the doctrine can come into play:
i. Court before which doctrine is invoked must be a proper one (pj & venue)
ii. Another proper forum must also be available

28 USC §1404a
1) Governs transfer from one proper forum to another proper forum, for the convenience of plaintiffs or witnesses in the interest of justice
2) District Court may transfer civil action to any other district or division where it might have been brought (not dismissal – rather transfer)
3) Hoffman v. Blaski (1960)
“Where it might have been brought” – means that the transferee district where action could have been brought in terms of good personal jurisdiction and venue, independent of defendant’s waiver or wishes.
4) Power of transfer exists only between federal courts, state courts can’t transfer
5) Court has choice of retaining or transferring – notion that everything is proper in the court’s own forum, can either keep the case or transfer it if it’s in the interest of justice or convenience of the parties and witnesses

28 USC §1406a
1) Governs transfer from improper forum to proper forum
2) Court has the choice of dismissing or transferring
3) Goldlawr v. Heiman (1962)
District Court may transfer case under 1406(a) even when no personal jurisdiction over D in transferor court as long as he’s transferring to a court where there is personal jurisdiction and venue

Piper Aircraft Co. v. Reyno
FACTS: Aircraft accident occurred and five passengers will killed, all Scottish citizens. Reyno represents the survivors of the deceased passengers and he filed in US b/c the laws regarding liability were more favorable than those in Scotland. Issue as to whether the action should be dismissed on the grounds of forum non-conveniens. Court ruled that forum should be in Scotland.

Stewart v. Ricoh
FACTS: Petitioner was an AL corporation who had an agreement containing a forum selection clause that provided that any dispute arising out of the contract could be brought only in a court located in Manhattan. Petitioner then brought a complaint in the District Court in Alabama. Issue as to whether a federal court sitting in diversity should apply state or federal law in adjudicating a motion to transfer a case to a venue provided in a contractual forum selection clause.
REASONING: Court held that must first determine whether there is a federal statute or rule on point. §14.04a governs transfer from one district to another, and it says the case may be transferred for the convenience of parties or witnesses when it’s in the interest of justice. Since §14.04a is a legitimate exercise of Congressional power under the Constitution and the forum selection clause should be considered within the context of applying the statute

Ferens v. Deere & Co
FACTS: Farmer was in an accident and brought tort claim in MS court b/c the statute of limitations had not expired. P then moved to transfer to PA court b/c it was a more convenient forum. Issue as to whether a transferee court must follow the choice of law rules that prevailed in the transferor court when a P transfers as they do when the D transfers under §14.04a.
REASONING: Court held that a transfer under §14.04a does not change the law applicable to a diversity case whether transfer initiated by P or D. Applying the law of transferor forum effects appropriate balance between fairness and simplicity.


Preclusion

PRECLUSION – What to say on the exam:
Look to see where Suit I and Suit II take place
If in different systems, mention FULL FAITH AND CREDIT DOCTRINE
Court must give full faith and credit to the judgment that the court in Suit I would give it.
Determined by the law of the system that rendered the judgment
Look to see if same parties or privies involved in Suit I and II
Claim preclusion only happens when you have the same parties in Suits I and II
1. Final judgment on the merits
2. Same cause of action? (Common nucleus of operative fact - Do the claims arise from same transaction or occurrence of as claims litigated in suit I)
If different cause of action, inquire about issue preclusion
Whether mutual (same parties but different C/A) or non-mutual (one of the parties is different), defensive or offensive (asserted defensively by defendant in Suit II or by plaintiff in Suit II), if it is issue preclusion, you must go through the 5 requirements for issue preclusion
1. Is the issue identical to the issue that was litigated in Suit I
2. Was the issue actually litigated, decided, and was the decision of that issue necessary to the judgment.
3. Was there a final judgment on the merits?
4. Party against whom plea was asserted was P or in privity with P to suit I?
You have to make sure that it is C on the Superbowl Sheet that is asserting estoppel. The non-party can come in and assert estoppel against someone who had their day in court. "You can't litigate this issue in Suit II, you already litigated it in Suit I. You had your shot."
5. Did the person in Suit I have the full, fair, opportunity to litigate?
In the exam, mention the rules quick. Then mention where the problem is.

Ø Claim Preclusion AKA “Res Judicata”
You get one bite of the apple .. so don’t F*#K it up….

Elements:
1) There must be a final judgment
2) The judgment must be “on the merits”
3) The claims must be the same in the first and second suits (Common nucleus of operative fact)
4) The parties in the second action must be the same as those in the first (or have been represented by a party to the prior action, or PRIVITY)

Goal: A party who has asserted a right to relief arising out of a particular transaction or occurrence must join all claims she has arising from it, or the omitted claims will be barred by res judicata.

Transaction or occurrence: Whether the party had the RIGHT to join the claim to the original action, whether they were AVAILABLE to the plaintiff in the first suit. Same "aggregate of operative facts as" as claim(s) asserted in Suit I.

Judgment on the merits: Dismissals for improper venue, personal jurisdiction were not litigated. A judgment is on the merits if the plaintiff had a full OPPORTUNITY to litigate the merits.

Opportunity: Say for example though, that a defendant fails to show up and the judge orders a default judgment for the plaintiff, the defendant had the opportunity to litgate on the merits.

Privity - One who has sufficiently close relationship to or identity of interest with party to Suit I to be subject to preclusive effect of judgment rendered in that action.

One who controlled the litigation (Insurance company who is paying the lawyer/providing the lawyer - controlling the litigation even though they're not named)

Successor in interest to property affected by judgment (Perhaps an heir or something)

One whose interests were adequately represented by party (there is some kind of legal relationship there/obligation to represent the interests. Fiduciary relationship, guardian)


Migra v. Warren City Sch. Dist. Bd. of Educ.
FACTS: Dr. Ethel Migra was unanimously re-hired for another year's contract in 1979. A few weeks later, the school board met again, minus one member, and voted not to renew the contract. Migra sued for breach of contract and wrongful interference by individual members of the board. The court then ruled for Migra. Later on, Migra was designated to fashion a voluntary plan to desegregate the schools and to develop a social studies curriculum. The board then rejected both plans and became hostile to Migra. Migra then filed these claims, mostly federal ones.
ISSUE: Whether plaintiff is precluded from litigating federal issue that could have been but was not actually litigated in Suit I in state court.
REASONING: The US Supreme Court says that they will follow Ohio State law under 28 USC 1738. Therefore they have to give the judgment from the state court the res judicata test since in this case the plaintiff is now filing federal claims in federal court based on the same transaction or occurrence. The federal claims COULD have been brought in state court. The key to learn from this case is that you need to RAISE ALL YOUR CLAIMS in the first suit. Even if they’re federal claims in a state court or state claims in a federal court. You might be precluded from doing so later.

Jones v. New England Life Ins.
FACTS:
Suit I (1993): FL Federal Court (removed)
P sued on policy: sought benefits + punitivesNE Counterclaimed for rescission and restitution
P lost on his claims
NE won on rescission and restitution

Suit II (1995)
Plaintiff is not happy, goes to state court in Georgia
Insurance Company removes to federal court.

REASONING: The Supreme Court found that they should follow Federal Rules in resolving this matter based on a diversity of parties suit. The 4 elements of res judicata are: 1) there must be a final judgment, 2) the decision must be rendered by a court of competent jurisdiction, 3) the parties or those in privity with them must be identical in both suits, 4) the same cause of action must be involved in both cases. In this case, 1 and 2 are obviously satisfied, 3 is satisfied because only NE Life is filing for summary judgment. 4 is satisfied based on the "Common Nucleus" test which is used for element 4. This is similar to the supplemental jurisdiction test in that the substance of these new claims are fundamentally the same as the claims from the state case in which the plaintiff has already lost. Therefore, the doctrine of res judicata applies and the case cannot be heard.



Ø Issue Preclusion AKA “Collateral Estoppel”

Elements:
1) Identity of issues
2) An issue that was litigated in the first suit and is being filed again (Litigated, decided and necessary to judgment in Suit I)
3) Final judgment on the merits in Suit I
4) Party against whom preclusion is asserted in Suit II was party or in privity with party to Suit I (The person having it asserted against must have been involved in the first suit)
5) Party against whom preclusion is being asserted must have had full and fair chance to litigate in Suit I.

Identity of issues
Suit I:
A v. B for infringement of patent Z
A loses, patent held valid

Suit II:
A v. C for infringement of patent Z
Issue of patent's validity is same

Make sure the issue was actually litigated (Identity of Issues)
Example
1. P v. D for breach of contract
D does not raise or litigate issue of lack of consideration going to enfoceability of contract
In suit for subsequent breach of same K by P v. D, D not precluded from litigating lack of consideration
The issue could have been raised in the first suit but it wasn't. It wasn't litigated!!!
(Actually decided) Rule: Where it's not clear from the record which matters were actually decided, and served as the basis for the judgment then there won't be any preclusion as to any matters. Where you can't tell what a jury actually decided, there won't be any preclusion

2. A v. B for breach of K
B raises lack of consideration and that A breached as a defense
Jury finds for B (No explanation - General verdict)
What was actually decided? Do we know?
We don't, but a court will not allow preclusion on these issues

3. P sues D for negligence
D denies negligence D raises contributory negligence of P as a defense.
Jury finds for D: general verdict
Maybe it was just D wasn't negligent, or they could have found that D was negligent, but P was contributorily negligent.

Necessary to Judgment
Example
1. A v. B for Breach of K
Jury finds good consideration for K
B wins based on jury's finding that A was in breach
Is finding as to consideration necessary to judgment for D?
B would have still won here. There won't be any preclusion. It was litigated and decided, but it wasn't necessary to the judgment that the jury reached in favor of B.

Suppose that the jury finds for A
B breached contract by not performing on time
B breached the contract by not delivering goods meeting specifications
Two Judgments (Maybe only one necessary) Rule: Where jury expressly finds for plaintiff on two distinct grounds, either of which would support verdict, some courts would give both findings preclusive effects

Some courts would give neither finding preclusive effect



Privity Example
A v. B (employee of C) For negligence
B is found not negligent
A v. C (employer) for B's negligence

Can C estop A on the issue of B's negligence? - YES


Bernhard v. Bank of America
FACTS: In June 1933, Mrs. Clara Sather made her home with Charles Cook in California. She soon went into a joint account with the Cooks. Later on, she deposited all her money into Cooks account. Cook then withdrew the balance of the account and opened a new account in the name of himself and his wife. A month later, Mrs. Sather died. The will beneficiaries sued the Cooks claiming they had no right to remove the money from that account. After a hearing, a probate court declared that Sather had made a gift to the Cooks. After Cook's discharge, Helen Bernhard was appointed administratrix with the will, and she instituted this claim against the Bank of America which holds the account.
REASONING: The three questions to answer are, 1) Was the issue decided in the prior adjudication identical with the one presented in the action in question? 2) Was there a final judgment on the merits? 3) Was the party against whom the plea is asserted a party or in privity with a party to the prior adjudication? In this case, the issue is the same, the order of the probate court settling the account was a final adjudication. The plaintiff brought the present action in the capacity of administratrix of the estate. In this capacity, she represents the same persons and interests that were represented in the earlier hearing. Therefore, C/E is available as a defense.

Ø Offensive/Defensive Nonmutual Use of Estoppel

Note – see the Superbowl Sheet for the diagrams on this one

Remember: Estoppel can only be asserted against a party that was in the previous suit, not against NEW parties. Also keep in mind PRIVITY.

Defensive Nonmutual Estoppel: Generally if the new party was a defendant who invoked estoppel to prevent the same plaintiff from Suit I from establishing a fact that the plaintiff had been unable to establish in the first suit. See Bernhard

Offensive Nonmutual Estoppel: Offensive use of collateral estoppel usually involves a new plaintiff who seeks to borrow a finding from a prior action to impose liability on a party who was a defendant in the prior action. See Parklane

Concerns about Offensive Use
Allowing offensive use does not promote judicial economy in same way that allowing defensive use does
Defensive use encourages Ps to join all potential Ds in first action.
Offensive use encourages "wait and see" attitude

Offensive Use may be unfair to defendant – PARKLANE FACTORS
1. Could P have easily joined in the first suit
2. Prior inconsistent judgments
3. Did D have incentive to fully litigate in Suit I
4. Procedural impediments
RULE: Where P could easily have joined in Suit I or where offensive use would be unfair to D, then don't allow it

Parklane Hosiery v. Shore
FACTS: Defendants were sued by the SEC for a proxy statement that was materially false and misleading. The SEC won. The stockholders have now filed a class action against the corporation and seek to collaterally estop the defendants from bringing this case to trial and relitigating the claims in a form of offensive collateral estoppel.
REASONING: The offensive use of collateral estoppel will likely increase the amount of litigation, and it may be unfair to the defendant. For these reasons, collateral estoppel may be allowed, but broad discretion must be given to the judges in order to prevent unfairness to the defendants. In cases where a plaintiff could easily have joined in the earlier action or where, either for the reasons discussed above or for other reasons, the application of offensive estoppel would be unfair to a defendant, a trial judge should not allow the use of offensive collateral estoppel. In this case, the plaintiff could not have joined in with the SEC, and the judgment in the SEC action was not inconsistent with any previous decision. The defendant had their chance to litigate the issue and they lost.


Ø Intersystem Preclusion


Full Faith and Credit Doctrine: Different statutes cover when the court in Suit II looks at which court made the judgment in Suit I.

State looking at another State - What effect state court should give to another state court. Article 4 Section I of US Constitution. State court determining preclusive effects of prior state court judgment must apply preclusion principles of forum that rendered judgment.

***State looking at Federal - 28 USC 1738 (Full Faith and Credit Statute) - Federal court, determining preclusive effect of prior state court judgment, must apply preclusion principles of forum that rendered judgment

Federal looking at State - 28 USC 1938 (Supreme Case decided this one) Stoll v. Gottlieb
- State court, determining preclusive effect of prior federal court judgment must apply federal preclusion principles (system that rendered judgment first should be applied in this new case)

Federal to Federal: Semtek v. Lockheed

In All 2 it is basically the same rule - The court in suit 2 must follow the rule from suit 1. It must give it the same preclusive effect that would be given by the system that rendered the judgment

Semtek Int'l Inc. v. Lockheed Martin Corp.
FACTS: Plaintiff brought a breach of contract and other tort claims against Lockheed in CA state court. The defendant removed based on diversity and then had the case dismissed based on statute of limitations. The plaintiff then tried to file in Maryland state court which has a longer statute of limitations. The court dismissed based on res judicata and the Appeal affirmed this.
REASONING: The plaintiff cited Dupasseur which stated that the res judicata effect of a federal diversity judgment is such as would belong to judgments of the State courts rendered under similar circumstances. Federal law is then not binding, but rather only states that a state must apply their law, or a fed court must apply state law of the state in which they sit in.
Rule 41(B) – Court concludes that effect of "on the merits" provision of Rule 41B is simply the barring of re-filing the same claim in the same federal court. Can’t be done in this district

Joinder of Claims/Parties

Rule 82: The rules below will not extend or limit Subject Matter Jurisdiction

Rule 20 – Joinder of Parties
(a) authorizes a plaintiff to sue multiple defendants or multiple plaintiffs to sue one defendant if:
1) They assert claims arising out of the same transaction or occurrence
2) Their claims against the defendant or defendants will involve a common question of law or fact.
NOTE: There is no requirement to sue multiple defendants or for plaintiffs to join together.

Rule 13 – Counterclaims and Crossclaims
(a) Compulsory Counterclaims – if the defending party’s crossclaim arises from the same transaction or occurrence as the claim against him, it is compulsory, which means you either assert it in the original action or lose it forever.
(b) Permissive Counterclaims – Completely unrelated to the original claim.
(g) Cross-Claims - A claim asserted by one party against a co-party, someone on the same side of the “v.” This is OPTIONAL, never required
(h) Permits joining additional parties to respond to Crossclaim or a counterclaim where such joinder would be required by rule 19 or 20
NOTE: If one asserts a cross-claim, then the response will be a COUNTERCLAIM.
NOTE: Under Rule 42(b) a federal court will almost always order a separate trial for a permissive counterclaim because it creates too much confusion for the court – all these claims and issues.
NOTE: Joinder of party who destroys diversity would be ok with respect to a compulsory counterclaim, or crossclaim, but not as to permissive counterclaim


Rule 18 – Joinder of Claims
(a) a party seeking relief from an opposing party may join with his original claim any additional claims he has against that opposing party. Unlike 20(a) there is no common transaction or occurrence requirement.
NOTE: Although no claim is required here, keep in mind res judicata and collateral estoppel.
NOTE: Any claims joined must also comply with subject/supplemental jurisdiction


28 U.S.C. 1367(b) - In any civil action in which district courts have original jurisdiction based on 1332 (DIVERSITY), the district courts shall not have supplemental jurisdiction over claims by PLAINTIFFS over persons based on rule 14, 19, 20, 24.
NOTE: A permissive counterclaim does not fall within supplemental jurisdiction under 1367(a) because by definition a permissive counterclaim is one that does not arise out of the same transaction or occurrence of the opposing party's claim
HOWEVER: If the amount in controversy on the permissive counterclaim was 80,000 (MA v. NY case with original jurisdiction on original claim then there's a counterclaim asserted) - YES because there is independent subject matter jurisdiction

Gutreuter v. Fiber Bond Corp.
Gs [KS] v. FB [IL/IN]
Now AT [KS] & Gs [KS] v. FB [IL/IN] (Compulsory Counterclaim)
Then Dralle [IL] & AT [KS] & Gs [KS] v. FB [IL/IN]

Dralle potentially destroys diversity

REASONING: To determine if a claim is compulsory the Court must evaluate the logical relationship between G's claims and FBC's counterclaims. A counter is compulsory if it speaks to the validity or essence of plaintiff's complaint. Count III for inducement of breach is compulsory. The question of count IV, is different because it alleges that Dralle sold products counter to FBC, and this wasn't proven, only they induced G to break out and form a new company. Therefore this count is permissive and tossed

Al-Cast Mold & Pattern, Inc. v. Perception Inc.
FACTS: Plaintiff filed for $17,000 in damages, and the defendant responded with a counterclaim for $50,000. The defendant then tried to remove to federal court based on diversity.
REASONING: The damages requirement of $75,000 must be from the plaintiff's complaint and not from any counterclaim or other defense move. Even aggregating the counterclaim and the complaint falls short, so the court has no SMJ.


Ø Impleader – Rule 14

Rule 14:
(a) Authorizes a defending party to bring into lawsuit any person not a party who is or may be liable to D for all or part of P's claim v. D.
(b) Authorizes a plaintiff to implead 3d party when Counterclaim asserted against P
NOTE: Rule 14 claims are NOT compulsory
NOTE: Also keep in mind that 1367(b) won’t allow supplemental jurisdiction for any claims brought by plaintiff against parties brought in by Plaintiff under Rule 14.

So always ask 2 questions
Permission (RULE 14?)
POWER - 1367??


Terminology:
3d party Plaintiff - original defendant
3d party Defendant - The third party guy being brought in by original defendant

Guaranteed Systems Inc. v. American National Can Co.
FACTS: Plaintiff (NC citizen) filed a state claim in NC against defendant (DE Citizen). Defendant removed and filed a counterclaim. Plaintiff then added on per Rule 14(b) a third party which is also an NC citizen on supplemental jurisdiction.
REASONING: Basically, under 1367(b), one cannot join on another party per rule 1367(b) if it was under diversity jurisdiction and using Rule 14. Before 1367 this would be allowed. After 1367 - NOT allowed


Ø Compulsory Joinder: Rule 19

19(A) - persons to be joined if feasible or necessary
(a)(1) - in that person's absence, the court cannot give complete relief to those already parties
(a)(2)Absentee has an interest relating to subject matter of the litigation such that disposition of the action without absentee may
(a)(2)(i) - as practical matter, impair/impede his ability to protect that interest
(a)(2)(ii) - prejudice those already parties by exposing them to substantial risk of double, multiple or otherwise inconsistent obligations

Rule 19(B) - Problems arise under Rule 19 when Absentee cannot be joined because not subject to PJ or joinder destroys diversity

Only when joinder of a R 19(a) party is not feasible must court turn to Rule 19(b) factors to decide whether action should go forward without Absentee or should be dismissed. If the action requires that the absentee party be present in the case, but it would destroy diversity or personal jurisdiction, then the case must be dismissed.

Very few people are indispensable

Rule 19(b) factors:
Prejudice to absentee or parties
Extent to which prejudice can be avoided by shaping relief
Judgment without Absentee adequate
Will plaintiff have adequate remedy if action is dismissed?

Professional Hockey Club of Central Sports Club of the Army v. Detroit Red Wings
FACTS: The Red Wings want the rights to Kozlov, and the Hockey Club claims they have rights. State claims have been filed between the plaintiff, an alien, and the Red Wings, a Michigan citizen. The Red Wings attempt to join Kozlov, an alien, and say he is an indispensible party
REASONING: Kozlov is a party who should be joined if feasible. He cannot be joined, however, because this would destroy diversity jurisdiction of the court. Therefore, the court must determine whether 'in equity and good conscience' this action should proceed with the named parties or be dismissed. It will be dismissed if Kozlov is indispensable and needs to be a part. The factors include 1) To what extent a judgment rendered in the person's absence might be prejudicial to the person or those already parties. 2) the extent to which by protective provisions in the judgment by the shaping of relief or other measure the prejudice can be lessened or avoided 3) whether a judgment rendered in the person's absence will be adequate and 4) whether the plaintiff will have an adequate remedy if the action is dismissed for nonjoinder. Kozlov has an interest in the litigation, but his interest will not be prejudiced if he is not joined and this fails factor one. Furthermore, the plaintiff having to try this case in a state court may be impartial due to the alien/local hockey club reasons.


Ø Intervention: Rule 24

(a)(2) - Intervention of Right -
1. Motion must be timely
2. Absentee must have interest in property/transaction that is the subject matter of the action
(a) Generally requires significantly protectable interest or substantial legal interest.
(b) Interest that is more substantial than a citizen - must be legal and standing
3. Absentee's ability to protect his or her interest may, as practical matter be impaired or impeded by adjudication without absentee
4. Absentee's interests are not adequately represented by the existing parties Generally where these cases - getting beyond interests - then number 4 is the stumbling block

In both cases we're dealing with state institutions and Affirmative Action
Hopwood Standard: Strong presumption that the interest will be represented by the government
Grutter Standard: even where government agency involved, burden in showing inadequacy is "minimal" need show only potential for inadequate representation

Rule 24(b)(2): Permissive intervention
Requirements: Absentee's claim or defense and main action have question of law or fact in common

Court must consider whether intervention will unduly delay/prejudice rights of original Parties. Totally discretionary - courts usually come up with a reason to deny it.

Hopwood v. State of Texas
FACTS: Two white students tried to sue the Texas Law School because their affirmative action in admissions was unconstitutional. Two organizations tried to join as defendants to the action, and the plaintiffs opposed the action.
REASONING: In order to show intervention by right, the intervening party must not adequately be represented by the existing parties. In this case, the only thing the state doesn't have is "evidence of past discrimination" which the court finds can simply be given to the state as defense since the state is adequately able to represent everything else.
Grutter v. Bollinger
FACTS: This case challenged the AA plans of Michigan and Michigan Law School and intervention by other groups
REASONING: Three things must be established in order to show that a party should have a right of intervention. 1) That they have a substantial legal interest in the subject matter of the case 2) that their ability to protect that interest may be impaired in the absence of intervention and 3) that the parties already before the court may not adequately represent their interest. The first element, probably the toughest to get around was met because the groups seeking to be enjoined are trying to keep admissions standards for their minority groups. The final two are met by burdens of proof that are minimal. The second element being that the minority students will be impaired in their ability to seek enrollment. Finally the University might possibly not represent the minority interests adequately because they're getting the admissions one way or another, so it isn't going to be as vigorously defended.

Cotter v. MAMLEO
FACTS: 3 police sergeants were promoted and only because they were black. Other white officers scoring the same test results sued based on discrimination and violation of equal protection. MAMLEO is a society seeking to help defend against these charges.
REASONING: Going through the 3 requirements: The group has a legal interest because all their clients are being advanced in this way. Also their ability to be promoted will be impaired, and there is possibility that they may not be adequately represented by the city of Boston.

Ø Interpleader: Rule 22

Two different ways to use interpleader: Rule 22 or 28 USC 1335
There are different ways to get the parties in, and they must be compared by looking at SMJ, PJ, and Venue.

Subject Matter Jurisdiction
1335
minimal diversity is ok.
At least two claimants who are diverse with adverse claims

Say you have Stakeholder from MA
Interpleads - 50 people from NY and one person from MA
This satisfies SMJ because there is one claimant from MA that is diverse from NY claim

Amount in Controversy only has to be $500 or more
Congress put a minimal amount of requirements on a federal case so that the federal court could hear a claim involving a lot of parties

Claimants would be the defendants if the stakeholder initiates it

Fund must be deposited


Rule 22
Federal Question or Diversity
Citizenship of stakeholder must be diverse from each claimant
Cannot have anyone on other side of V with same citizenship
Amount in Controversy must exceed $75,000 Measured by the amount of the stake


Venue
1335
Under Section 1397 - Any district where one or more claimants reside
Need not all reside in same state

Rule 22
1391(a) - District where any defendant resides if all in same state or district where events giving rise to claim occurred or property located


Personal Jurisdiction
1335
2361 authorizes nationwide service of process in statutory interpleader cases
Recall rule 4(k)(1)(c) - Service Process = personal jurisdiction -

Rule 22
Regular principles
Long Arm, minimum contacts


The statutory requirements are much more liberal
If you had a choice - always go for statutory interpleader - but there are times when you cannot use the statute -
When all the claimants are from the same state - you cannot use the statute

Franceskin v. Credit Suisse
FACTS: Franceskin has brought state law claims against CS and Lloyds. Franceskin is a citizen of Argentina and CS and Lloyds he claims to be citizens of NY.
REASONING: CS is an alien because it is also a citizen of Switzerland. Lloyd is also an alien so there is no SMJ

Class Actions: Rule 23

Must have an ascertainable class – Use a 23 A test
All people who have a common law or issue.
Class representative must be a member of the class
Must satisfy 23(a) factors
Numerosity - Over 40 people? Enough that doing individual trials or joinder would be a problem
Commonality - Common questions of law or fact, common issues relating to class
Typicality - The claims and defenses must be typical of the class
Adequacy of Representation - You must make sure that the class representative is adequate, adequacy of class counsel

23(g) - now addresses appointment of class counsel (Representation)
· The work that this counsel has done
· Experience of counsel
· Knowledge of applicable law
· Resources that counsel will commit to litigation

Hansberry v. Lee

Suit I: Burke v. Kleiman
· B for herself and other owners in the area
· Defendants = buyers of property formerly owned by signers of covenant - NOT A CLASS
· Stipulation that 95% of the owners have signed, meeting one of the requirements
· Restrictive covenant upheld
Suit II: Lee v. Hansberry
· Suit to enjoin breach of covenant
· D argues covenant not effective because not signed by requisite 95%
· Lee comes back with res judicata that it had already been decided
· No appeal taken in Burke v. Kleiman case - it wasn't hotly contested
· Supreme Court of IL affirms the judgment even though 95% had not signed
· Hansberry was bound by Burke v. Kleiman - the S Ct of IL is that Burke brought the suit on behalf of herself and others subject to the covenant. Since Hansberry is a member of the class, they are bound by the judgment
Now the case is in the US SC
The general rule is that a party is not bound unless they were a party to the litigation or in privity with a party
The class or representative suit is a recognized exception to the "party requirement"
The SC recognizes that if they were members of a class and the issue was litigated, then the class could be bound
The concern is of DUE PROCESS
D/P requires procedure that fairly insures protection of interests of Aees to be bound by judgment
Was there a problem in the state court suit?
Lack of adequate representation in Suit I precludes binding effect of that judgment on Hansberry petitioners



Types of Class Actions
23(b)(1) - prejudice class
23(b)(1)(A) - concerned about prejudice to party opposing class - a defendant may be subjected to inconsistent obligations if individual suits are allowed to proceed - one court may order some kind of performance, another court may order something different
23(b)(1)(B) - concerned about prejudice to Absentee (putative) members of the class
23(b)(2) - interest in uniformity of enforcement - easiest to recognize
· The defendant's conduct is generally applicable to class
· Relief sought = injunctive/declaratory relief (NOT DAMAGES)
23(b)(3) - damage class
This is where we get into what happens when some of the members don't have the requisite amount in controversy…etc
· The common questions of law or fact must predominate in this case
· Class action device must be superior method of adjudication
Often, in 23(b)(3) actions, issue of liability = common but issue of $ may require individual treatment

23(c)(4)(A) allows certification as to particular issues
Notice required in 23(b)(1) or (2)? It's PERMISSIVE – not required
23(c)(2)(A) says that court may direct appropriate notice to class
It isn't always required - there'll be some kind of notice, but in a (b)(2) class - you may not get any formal letter - maybe there'll be some bulletin board that it's up to you to see.

Notice in (b)(3) class -
REQUIRED
23(c)(2)(B): mandates best notice practicable under the circumstances, including individual notice to all members who can be identified through reasonable efforts. This is only for B3 Classes
Notice must also be in PLAIN LANGUAGE

Notice must inform as to:
Nature of action/definition of class/class claims
Right to make appearance through counsel
Right to Opt Out
Binding effect of class judgment

In a B1 or a B2 suit - there is NO OPTING OUT - once you are determined to be a member of the class, you're bound by the judgment whether you like it or not

23(c)(3)
· Judgment in (b)(1) or (2) shall include and describe those court finds to be members of class
· Judgment in (b)(3) shall include all those to whom notice was directed who did not opt out and whom court finds to be members of class

Settlement 23(e) - Class actions cannot be dismissed or settled without court approval
Notice of any proposed dismissal or settlement must be given to all class members
NEW! - if (b)(3) class, court may order that another opt-out opportunity be provided before settlement approved

Mack v. Suffolk County
FACTS: The plaintiffs seek to enjoin in a class action, all women stripsearched over a 3 year period. They seek to be noticed as a class and this is the motion to do so.
REASONING: This must pass the Rule 23(a) test of numerosity, commonality, typicality and adequacy of representation. In this case, the class is every female who was subjected to the stripsearch which is way to big to track down all at once. The question of law is all the same, is a mandatory stripsearch legal, the typicality is also the same, and adequacy is also fine. Finally, there is 23(b)(3) certification, that the common questions predominate individual questions. In this case, the common question is what was mentioned before, this is the most efficient way to litigate the case.

Class Certification from Mack v. Suffolk County
Numerosity: estimate several thousand women subject to policy between December 1995 and December 1998
Common questions of law or fact
Claims not in conflict with one another
Claims need not be identical
May be varying defenses as to particular individuals
Adequacy of Representation
No showing of intra-class conflict
Counsel - experienced civil rights attorney

Requirements of 23(a) satisfied
(b)(2) certification appropriate

The more difficult question was the b3 certification -
Look at:
Predomination of common questions over individual questions?
Superior Method of Adjudication
Defendants argue multitude of mini trials will be required on liability and damages
Liability likely to be uniform
Diversity? How is it determined?
· Amount in Controversy?
§ Supreme Tribe of Ben-Hur (1921) - look at citizenship of named representatives of the class - not all the class members
§ Snyder v. Harris - Separate and distinct claims of class members cannot be aggregated to meet amount in controversy. Every member must satisfy the requirement
§ Zahn v. Int'l Paper Co. 1973 - Each plaintiff in a (b)(3) action must satisfy the amount in controversy requirement - NO PENDENT JURISDICTION
Ortega v. Star Kist Foods, Inc.
FACTS: Girl cuts her hand on some tuna, she sues and her family sues for emotional damages.
REASONING:
Why federal court for "pinky cut"
· Not a class action, but rather can't get a jury trial
Joinder of plaintiffs under Rule 20
Diversity of citizenship?
· Amount in Controversy
· Daughter DOES satisfy the amount
· Parents' and sisters' claims for emotional distress do NOT
· Plaintiff only need show that it is not a legal certainty that the claims will not result in verdict for amount in controversy
· Question then as to supplemental jurisdiction for plaintiffs joined under rule 20
· Court reserves question of supplemental jurisdiction in class action context
· Precedent (TROY BANK 1911) and Clark (1939) - "when two or more plaintiffs having separate and distinct demands united for convenience and economy in a single suit, it is essential that the demand of each be of the requisite jurisdictional amount
· Does 1367 now overrule Clark?
· How does court read 1367A?
· Unless strawbridge and clark are satisfied, the diversity statute does not confer original jurisdiction on the district cout
· If original jurisdiction requirement of 1367(a) not satisfied, then no supplemental jurisdiction cannot attach
· Flaws with such construction?
· Not consistent with 1367b's exclusion of claims by plaintiffs deemed indispensable under Rule 19
· Consistent with exclusion of claims by Ps against parties joined under rule 20?
· Not consistent with policies of efficiency/economy
· Court notes: Under contrary view: Strawbridge overruled by 1367(a)
· NY +MA v. NY
· Assuming Amount in controversy satisfied by MA v. NY claim
· Assuming claims are related
· Supplemental jurisdiction would then be OK here too, but there's no diversity, there's no basis …??????
· What does the legislative history of the statute suggest?
· The courts that are saying that Zahn is overruled, and saying that if the statute is clear, they don't read history.
· If statute on its face is clear, no need to look at the history

Ortega v. Star Kist - NOT A CLASS ACTION
Joinder of plaintiffs under Rule 20
Beatriz's claim satisfied amount in controversy (she was the little girl)
Parents' and sister's claims did not

Rule 20 joinder problem - question then as to supplemental jurisdiction for plaintiffs joined under Rule 20
Note court reserves question of supplemental jurisdiction in class action context.
New plaintiffs against a party that wasn't asserted under rule 20. The plaintiffs were joined under Rule 20

Clark 1939 - when two or more plaintiffs having separate and distinct demands unite for convenience and economy in a single suit, it is essential that the demand of each be the of the requisite jurisdictional amount.

Does 1367 OVERRULE Clark???

1st circuit case
Unless Strawbridge and Clark are satisfied, the diversity statute does not confer original jurisdiction in the district court.

Court notes: Under contrary view Strawbridge overrurled by 1367

NY + MA v. NY
Assume amount in controversy is fine, claims are related
Supplemental jurisdiction?

If 1367a has overruled clark and strawbridge, this is OK
The 1st circuit thinks it's a big deal to overrule strawbridge

Allapattah - class action by 10,000 exxon dealers v. exxon
Dist certified for appeal - can supplemental jurisdiction be used for non-members of class because they didn't meet amount in controversy

Diversity? How is it determined?
· Amount in Controversy?
§ Supreme Tribe of Ben-Hur (1921) - look at citizenship of named representatives of the class - not all the class members
§ Snyder v. Harris - Separate and distinct claims of class members cannot be aggregated to meet amount in controversy. Every member must satisfy the requirement
§ Zahn v. Int'l Paper Co. 1973 - Each plaintiff in a (b)(3) action must satisfy the amount in controversy requirement - NO PENDENT JURISDICTION
Question in Allapattah: Whether Zahn has been overruled?

Due process requires: - See Notice
Notice and opportunity to participate in litigation
Opportunity to OPT OUT
Adequate representation

Pleadings
Pleadings set forth position of parties on claim
Documents submitted to court/other parties setting forth a position
Motions = applications to court for an order
Rules 7-15
Rule 7(a) - identifies pleadings as complaint, answer, reply to counterclaim, answer to cross-claim, 3d party complaint, 3d party answer
Court may order a reply to an answer or third-party answer

Rule 8(a) - complaint must state grounds for court's SMJ
Complaint should contain a short and plain statement showing pleader and reason (entitlement) for relief
Demand for judgment

Conley v. Gibson 1957
Ds move to dismiss on 3 grounds
Nat'l RR board had exclusive jurisdiction
Failure to join railroad on rule 19
Failure to state claim on which relief could be granted

Lower courts dismissed for want of SMJ
S Ct reverses

Ct address on other grounds
RR not indispensable party
No effect on RR's rights on interests by enforcement of duty of fair representation - old rule - a party that would be bound by the litigation

Test? <---------IMPORTANT STILL USED TODAY
Complaint should not be dismissed for failure to state claim unless it appears beyond doubt that P can prove no set of facts in support of his claim which would entitle him to relief.


All that is required by Rule 8(a)(2) is short and plain statement of claim that will give defendant fair notice of plaintiff's claim and grounds upon which it rests

"notice pleading" is what we call this

Swierkiewicz v. Sorema (2002) - Rule 8(a)'s simplified pleading standard applies to all civil actions, with limited exceptions

Rule 9(b) for example, provides for greater particularity in all averments of fraud or mistake
Court has declined to extend such exceptions to other contexts


Davis v. Ruby Foods

Title VII, sexual harrassment claim
P bringing suit pro se
Defendants motion to dismiss for failure to comply with Rule 8 - granted (by lower court)

Problems with the complaint - Issue- whether the district court abused its discretion in dismissing complaint merely because it contained repetitious and irrelevant matter
A disposable husk around a proper core of proper pleading
Court Notes: Complaints signed by lawyers are never dismissed simply because they are not short, concise and plain
Dismissal of complaint if unintelligible is unexceptional

Excess can be ignored

Court notes exceptions (e.g., 240 page complaint in 3d circuit case)
Excess burden in this case created by Defendant's motion to dismiss action of district court in granting motion

Court advises Defendant against moving to strike extraneous matter unless actually prejudicial to defendant


Eison v. McCoy (7th Cir. 1998)
Amending Complaints
June 17, 1996 - P files Complaint
Alleging police misconduct on 2 different occasions
June 16th 1994 - 1sst incident
Dec 16th 1995

2 year S/L - satisfied by original complaint

Problems:
1st complaint named officers by nicknames - TC, Pac Man, Somethingman and Crater Face
On July 24, 1996 - Amended Complaint substitutes proper names
Clearly after the 2 year S/L for 1st Complaint

3 officers move to dismiss

District Court dismisses all claims in Count I (June 1994 incident) as barred by S/L

FRCP 15(c)(3) not satisfied

What does Plaintiff Argue? - Original complaint adequately identified defendants
Amended complaint was merely clarifying, curing misnomer
2. Also argues that if this is a change, 15(c)(3) was satisfied

Court's response -
This isn't just a misnomer, where proper party is before court, merely correcting name under which sued (We have ABC Corp, but you called them ABC Limited) - that's a misnomer
Here, no one knew who these parties were

No mistake here, simply no knowledge as to proper identity of defendants
No relation back

District Court erred in not allowing amendment naming Jackson as Defendnat (pac man) on Count II ('95)
The amended complaint was filed within 2 year period so it was fine

Note: 15(c)(1) - amendment relates back to date of original pleading when relation back is permitted by law that provides applicable statute of limitations


Ø Motion to Dismiss: Rule 12(b)
12(a) – Answer – when it should be presented
12(b) – Pre-Answer Motion - Why it should be dismissed - Optional
1. Subject Matter
2. Personal Jurisdiction
3. Improper Venue
4. Insufficiency of process
5. Insufficiency of service of process
6. Failure to state claim for which relief can be granted (On the merits)
7. Failure to join a party under rule 19
12(e) – Motion asserts that the plaintiff’s claim is so incomprehensible that the responding party cannot meaningfully respond to it.
12(g) – If you have multiple 12(b) motions, you must use all the ones you want to raise in one pre-trial motion, or lose the rest.
12(h)(1)(A) – 12(b)2-5 are waived if not raised in a timely manner (before litigation)

Basically – if you make a 12(b) defense – use all that you can, because 2-5 will be barred from future defenses if you don’t raise them while raising anything under 12(b) pre-trial motion.


Ø Rule 11 Sanctions
Rule 11 (a) - requires every pleading, written motion or other paper to be signed by attorney of record (or by party if no attorney)
Rule 11 (b) - By presenting to court (by signing, filing, submitting, or later advocating) a pleading, written motion or other paper, attorney certifies that … after inquiry reasonable under circumstance … You're certifying that you've done some looking into these statements you're putting before the court.
1. It is not presented for any improper purpose; e.g., to harass or cause delay
2. Claims, defenses & other legal contentions are warranted by existing law, or by nonfrivolous argument for extension, modification, or reversal of existing law or establishment of new law
3. Allegations have evidentiary support or likely to have support after discovery and
4. Denials of factual contentions warranted by evidence
Rule 11 (c) - If, after notice & reasonable opportunity to respond, court determines that 11(b) has been violated, court may impose appropriate sanction on attorneys, firms or parties in violation
Rule 11 (c)(1)(A) - Requires separate motion for sanctions to be served on party, who then has 21 days to take advantage of "safe harbor" provision. Correct whatever has been done that violates 11(b)
Rule 11 (c)(1)(B) - Court on its own initiative, may issue order to show cause why party has not violated 11(b)
No safe harbor provision

Rule 11 (c)(2) - Primary purpose = deterrence, not compensation; $$$ ordinarily paid into court; moving party may get fees & expenses incurred as direct result of violations
Rule 11 (c)(2)(A) - sanctions not to be awarded against a represented party for violations of 11(b)(2): frivolous contentions of law
Rule 11 (c)(2)(B) - Where court acts on own initiative, can't impose sanctions if there is a voluntary dismissal or settlement of action before court issues order to show cause
Rule 11 (c)(3) & 11(d) - C3 requires district court judges to describe conduct and basis for sanction imposed
Rule 11(d) - makes clear R11 not appropriate for discovery abuses, that's Rule 26


Discovery

Rule 26 - gives you the thrust/scope of discovery and gives doctrines of what will be applicable. Sets out the forms discovery might take and the methods of discovery

Rule 37 - the place you go when there is a problem - the other side is not complying with a discovery request

26(a)(1) - Except in categories of proceedings specified in rule 26(a)(1)(E) or to extent otherwise stipulated or directed by order, party must automatically disclose certain info without discovery request.
Identity of individuals likely to have information that disclosing party may use to SUPPORT its claims or defenses
Documents to same effect
Computation of damages along with documentation supporting this
Insurance agreements

26(a)(2): Disclosure of identity of expert witnesses; written reports by experts specially retained for the case

26(a)(3) - pretrial disclosure of info regarding evidence that may be used at trial; witnesses, documents, depositions, etc…

26(a)(5) -
1. Depositions (oral/written) - schedules made - can be party or witnesses
2. Interrogatories - 25 limit - WITNESSES ONLY - from here on
3. Production of documents/things
i. 1 day, 7 hours, 10 depositions - MAX unless court order allows further
4. Physical/Mental Exams
5. Requests for admission

26(b)(1) - Allows discovery of any matter, not privileged , that is relevant to the claim or defense of any party
Facts are discoverable if:
1) Relevant to subject matter of litigation
2) Not privileged
3) If not themselves admissible, then reasonably calculated to lead to admissible evidence.
26(b)(2) - by order, court may later limits on number of depositions and interrogatories, length of depositions under Rule 30
By order or local rule, court may limit number of requests under Rule 36.

26(b)(2)(i) – If discovery is obtainable by a cheaper source, etc
26(b)(2)(ii) – The party seeking discovery has had ample opportunity by discovery to obtain information sought
26(b)(2)(iii) – the burden or expense outweighs the likely benefit, taking into account the needs of the case, the amount in controversy, the parties’ resources, the importance of the issues at stake, etc.

26(b)(3): Work product of attorneys
26(b)(4): EXPERTS
26(b)(5): requires party withholding info based on privilege or work product protection to make claim expressly

26(c): provides for protective orders, but must certify that you attempted to confer with the other party – such as excluding expert testimony.
26(e): requires (a)(1) and (a)(2) disclosures to be supplemented if party learns info incomplete or incorrect in some material respect.
26(f): parties required to meet and discuss possibilities of settlement and formulate discovery plan which is to be submitted to court
26(g): signing of disclosures requests, responses and objections = certification to cout that disclosure is complete and correct at time made and ….

Physical Mental examinations:
Sibbach v. Wilson (1941) - can you order a party to submit to these exams?
The court resolved the question with respect to a plaintiff - upheld constitutionality where plaintiff in negligence action asserted physical injury as basis of recovery.


Ø Expert Witnesses
26(a)(2)(A): Mandatory disclosure of identity of any person who may be used at trial as expert Witness
26(a)(2)(B): Disclosure must include detailed report by any expert retained or specially employed to provide testimony in case
Publications, books, anything that qualifies a person as an expert - must be included

Experts: Rule 26 REPORT
· Report must cover opinions to be expressed and basis for opinions including data or other information considered by W in forming opinions, and provide substantial details about qualifications and experience as a witness

26(b)(4)(A): authorizes deposition
· What the expert will testify to
· All that goodness
· Deposition of expert required to prepare report may be taken only after report served

26(e)(1): requires disclosure of any material changes made in opinions of expert from whom report is required

26(b)(4)(B) - deals with expert retained or specially employed by another party in anticipation of litigation or preparation for trial who is not going to be called as Witness at trial
If expert gives opinion other than what you want - you can't force other side to give over info of experts they've consulted who WON'T be used at trial
26(b)(4)(B): to make discovery of facts known or opinions held by such expert, party must show exceptional circumstances under which it is impracticable to get info by other means

Majority of courts agree that disclosure of fact work product to testifying expert, in conjunction with expert's consideration of those materials, results in waiver of work product immunity. Following 26(a)(2)(B) – ANY type of material, from a non testifying expert given to a testifying expert, this is all waived when it’s given to a testifying expert. Even if not part of the opinion, if it was looked at, it’s waived.

Ø Work Product: Rule 26(b)(3)

Hickman v. Taylor Note: This was before Rule 26(b)(3) was in place

The facts: Tugboat sinks - 4 people die, end up with one plaintiff who is pursuing the case
The problem: The plaintiff is trying to get defendant's attorney to turn over certain pieces of information
· Testimony from surviving members (statements recorded by lawyer) and whatnot
· Written statements signed by survivors
· Memoranda, attorney puts together, a product of oral interviews
Tangible documents and things prepared in anticipation of litigation by or for another party is called ORDINARY WORK PRODUCT

Rule on ordinary work product -
Discoverable only upon a showing of substantial need and an inability to secure substantial equivalent of items through other means without undue hardship
The protection is for the documents, not the facts. The facts may be discoverable in another way.

Note: materials assembled in ordinary course of business or pursuant to public requirements unrelated to litigation, or for other, non-litigation purposes are not protected by 26 (b)(3)

RULE ITSELF: Expands on Hickman by protecting material prepared by attorney and materials prepared by or for a party or by or for that party's rep (i.e., documents prepared by insured, agent, etc)

Second category of info sought in Hickman memos relating mental impressions of attorney based on oral interviews conducted with witnesses
"Core" or "Opinion" work product

In Hickman, Court suggests one could probably never make showing sufficient to require attorney to turn over report based on oral interviews and mental impressions derived ….

Ø Privileged Matter

Attorney – Client Privilege – communications with an attorney, the substance of which are privileged will be barred from disclosure under the opinion work product. 26(b)(1) protects privileged information

Upjohn v. United States
FACTS: There was some bribing going on in the European section of a US company. The US CEO decides to send out a questionnaire to all European employees about the matter as an internal investigation. The IRS gets wind and wants these documents too.
REASONING: The IRS can interview the employees, but they don’t have a right to the documents because those are protected by attorney client privilege. The facts are not protected, but rather the documents under opinion. Therefore, go after the facts yourself.

United States v. Adlman
FACTS: Adlman decides it wants to combine some subsidiaries so that it can cut costs and take a nice tax break. They also know that something like this is going to make the IRS come calling. The question decided is whether a study prepared for an attorney assessing likely result of expected litigation is ineligible for 26(b)(3) protection if primary purpose of making study was to assess desirability of a business transaction, which, if undertaken, would give rise to litigation.
REASONING: The 2nd Circuit calls this work-product. It was created by or for a party in anticipation of litigation. It lays out strategies and defenses and therefore will be protected.

Ø Electronic Discovery

Prior to this, there was an 8 step test - previously the state of the law: (determining who should pay)
Extent to which the request is specifically tailored to discover relevant
Availability of such information from other sources
The total cost of production compared to the amount in controversy
The total cost of production compared to the resources available to each party
The relative ability of each party to control costs and its incentive to do so
The importance of the issues at stake in the litigation
The relative benefits to the parties of obtaining the information

Summary Judgment

Summary Judgment Chart:
When it may come up:

Rule 12(b)(6) - Complaint - Motion to dismiss (sometimes for failure to state a claim)

Rule 12(c) - Pleadings - Look at Complaint and Answer - (Motion for a judgment on the pleadings)

Rule 56 - Discovery - This is a put up or shut up phase - you need to come in with any documents you've received. Enough evidence must be put in from which a jury could find in favor of the plaintiff on the essential elements of the plaintiff's claim. If there's no way a reasonable jury could find for the plaintiff based on the evidence put before the court, this would be a case ripe for summary judgment.
Are there any genuine issues of material fact which will go to a jury?

Rule 50(a) - Evidence - Now they go to trial and plaintiff comes in and puts out their case. Now the court is looking at everything, pleadings, discovery, testimony. After plaintiff puts on their case, the defendant may make a motion for a judgment as a matter of law (Directed Verdict) - At this point, we're up to the point where there is even more information. The defendant is saying "plaintiff hasn't put in enough for which a jury could find for the plaintiff's favor.

Rule 50(b) - Jury Verdict - Renewed Motion for a judgment as a matter of law. Essentially what the defendant is saying is that there wasn't enough to send to the jury in the first place. Reconsider the decision of having sent this case to the jury and find that I am really entitled to a judgment as a matter of law.

Ø Rule 56

Summary Judgment - Rule 56 - party seeking summary judgment must show that there is no genuine issue as to any material fact and that party is entitled to judgment as a matter of law.

When party moves for summary judgment, the party generally will accompany motion with affidavits which must satisfy standards set forth in 56(e).
· Must be made on personal knowledge
· Must demonstrate competence to testify
· Must set forth facts admissible in evidence
· In addition to affidavits, Rule 56(c) permits court to consider products of discovery … admissions, interrogatories, depositions, documents, etc.
· Note that both a 12(b)(6) m/t/d and a 12(c) motion for judgment on the pleadings will be converted to a motion for summary judgment if matters outside pleadings are considered by court.

Celotex Corp. v. Catrett
FACTS: Plaintiff filed suit claiming death of husband was from exposure to asbestos. There were no witnesses who could testify as to plaintiff's exposure. Plaintiff didn't put in anything linking their product to the death of the husband. LACK OF EVIDENCE. The defense moves for summary judgment, but puts up no proof of their own. They just say that the plaintiff didn’t prove anything at all.
REASONING: A party may move for summary judgment WITH or WITHOUT supporting affidavits. The rule does not require this. "Where nonmoving party has burden of proof on material fact, summary judgment motion may properly be made in reliance solely on pleadings, depositions, answers to interrogatories and admissions on file."

Anderson v. Liberty Lobby, Inc.
FACTS: - Magazine - investigator printing an article that was defamatory purporting the Liberty Lobby to be racist, sexist, nazis, etc….. Defendant moves for summary judgment - basis for the motion - on the law of libel, when a public figure is involved, the burden of proof in a libel case is clear and convincing evidence of actual malice before the plaintiff can recover. A motion for SJ is filed by defendant and granted in lower court.
REASONING: To defeat a motion for summary judgment, the plaintiff must show a genuine issue of material fact. A material fact is one that is going to make a difference in the outcome under governing substantive law.
Judge must direct verdict if, under governing law, there can be but one reasonable conclusion. Primary difference between summary judgment motion and the motion for directed verdict is the timing
Summary Judgment: Before trial, documentary evidence, products of discovery
Directed Verdict: At trial, evidence and testimony.
Jury Trials
Ø Rule 58

Rule 28(a) preserves the right to jury trial as declared by the 7th Amendment or as given by a statute of the United States. 7th Amendment (1791) "In suits at common law, where the value in controversy shall exceed $20.00, the right of trial by jury shall be preserved."

Question: When there are multiple issues in a suit, both equitable (perform or stop performing) and legal, which ones are to be decided first?

Beacon Theaters v. Westover
FACTS: There was an equitable claim being asserted, and a legal counterclaim then being asserted after that.
REASONING: If issues of fact are common to both legal and equitable claims raised in the same action, and jury trial has been demanded on issues material to the legal claim, issues must be decided by jury before equitable claim decided by judge.

Situations where Beacon Theatres applies:
1: Claim/Counterclaim situation
2: Plaintiff seeking both legal and equitable relief (Dairy Queen case)
3: Plaintiff seeks legal or equitable relief (E.g. specific performance or $$$ for breach of contract)

Dairy Queen Case
FACTS: Plaintiff sued for a breach of contract and violation of copyright. Plaintiff seeking an injunction versus future use of its trademark and franchise and accounting of profits since breach.
REASONING: Jury trial right attaches to legal issues whether incidental to equitable issues or not.

Highlights the Ross v. Bernhard (1970) case: Ross was a stockholders' derivative action v. directors and corporate brokers. The relief sought was $$$ damages. At common law, stockholders were not permitted to sue corporate managers at law. It had to be brought in EQUITY>

If the corporation were suing under its own name and seeking legal relief. That could be brought at law.
Question: How should this stockholders' derivative suit be treated post-merger?
Holding: Purely procedural impediments which prevented certain types of action from being heard at law are no longer controlling under merged system of law and equity.

Therefore, though an action may be brought before the court by means of a device recognized only in equity, if such action presents legal claims or issues, then right to jury trial attaches.

Note: Also keep in mind that the claims need to be brought up in the same suit. As in that Parklane case, where the judge decided the issues of fact, that did not need to go to a jury. The reason why if both legal and equitable claims are brought up, the legal should go first is that a judge’s interpretation of facts for the equitable claim may have an effect on the jury’s opinion.

Ø Special Verdicts: Rule 49 + 50

Rule 49(a)
It gives court authority to dispense with general verdict and to submit various fact issues to the jury in the form of special interrogatories, on each of which jury returns a special verdict.

Rule 50(a)(1)
Court can grant motion for JMOL at any time during trial, as soon as it's apparent that party has no legally sufficient evidentiary basis from which jury could find for party on issue that is essential to party's case

Rule 50(a)(2)
Motions for JMOL must be made prior to submission of case to jury
Must articulate basis for motion so that nonmoving party has opportunity to cure deficiency before case goes to jury.

Rule 50(b)
If motion for JMOL is made at close of all evidence and denied, court is deemed to have submitted case to jury subject to later resolution of legal question raised by motion.
· Judge will let it go to jury. If jury comes back with a verdict he doesn't feel they could have found, the judge might reverse it
· If you get a jury verdict and the judge says "on the legal question, I don't think there was enough evidence here for which they could find for the plaintiff, after the verdict, I'm disappointed they came back this way, but I'm going to rule JMOL for defendant"
· Plaintiff takes appeal
· The appellate court says "we disagree with you lower court, we think there was enough evidence for which a jury could enter the verdict for plaintiff.
· If the court didn't let it go to the jury - before the case goes to the jury, I'm going to grant JMOL and plaintiff takes an appeal, and the appellate court says "we don't think so" - then you have to RETRY the case … ENTIRE case. That jury is gone - new jury, everything, because you need a jury’s verdict

Legal Question: is there sufficient evidence from which jury could find for nonmoving (generally the plaintiff) party on material issues
Question: what evidence is considered in making this determination?
Federal Standard: Consider all evidence in light most favorable to non-moving party, (generally, plaintiff) plus any evidence most favorable to moving party that is not impeached or contradicted


Robles-Vazqyez v. Tirado Garcia
FACTS: The plaintiff is the common law wife of a guy shot while trying to escape in Puerto Rico - Missing his payments. This woman is suing on behalf of her common law husband's estate for his injuries and also for her own behalf and children's for their pain and suffering. Claims brought under 4th Amendment and also under Puerto Rico Constitution. The defendant moves for JMOL - District Court Denies. Jury comes back and finds for Ps (estate of deceased and for Plaintiff's own behalf (children and wife)) District Court granted JMOL after the jury verdict
REASONING: What is the rationale underlying 50(a)(2) requirement that motion specify law and facts on which motion is based prior to submission of case to jury? It needed to be raised BEFORE the case went to the jury. Neither trial court nor appellate court can consider issue raised for first time in post-verdict motion for renewal of motion for JMOL.

Ø Grant a New Trial – Rule 59

Rule 59 allows judge to grant new trial, either on motion of party or on its own initiative

Motion must be made within 10 days after entry of judgment
Grounds for granting new trial:
1. Prejudicial judial error, Misconduct of court, parties or counsel
2. Newly discovered material evidence (not available, or easily discoverable - something that just happened)
3. Jury Misconduct
4. Verdict = against wt of evidence ***
5. Inadequate or excessive damages ***

Roy v. Volkswagon
FACTS: Product liability suit arising from single car accident. Jury returned 3 million dollar verdict for plaintiffs. Found defect in design of VW van and this was the proximate cause of Ps' injuries. Distr. Court set verdict aside and orders a new trial. VW asks for a new trial because they thought the original verdict was against the weight of the evidence, this is granted and VW wins. Then Roy appeals the verdict.
REASONING:

Why does Ct. of Appeals conclude Dist. Ct. erred in granting a new trial in the first place?
· If a court thinks a jury's verdict is the product of sympathy or prejudice - they come out with a verdict based on the injuries and they ignore evidence in defendant's favor, that may be legitimate grounds
· That may be the case here, but they were instructed not to let that affect the decision and there is no evidence that this was the product of passion or sympathy
· There was a lot of testimony by experts on both sides. This verdict was not so against the weight of the evidence

Ø Remittitur and Additur - Rule 59

A motion under R59 is an appropriate means to challenge the size of the verdict

Judge has the power to order a new trial whether the verdict is unreasonably high or low.
Where verdict found to be excessive, court may condition its denial of D's motion for new trial upon Ps consent to a remittitur in a stated amount. Basically if plaintiff is willing to lower amount - they can sort of settle (remit).

District court cannot say "I'm reducing the award" they have to give plaintiff the option "Accept this or I’m ordering a new trial"

Federal courts may grant new trial because verdict = inadequate, but NO ADDITUR IN FEDERAL COURTS
They can only order a new trial.

Some states permit additur (59A) Massachusetts does
Where verdict inadequate, court conditions denial of Ps motion for new trial on Ds consent to entry of judgment for stated amount in excess of verdict.

Appeals

28 USC 1291: Courts of Appeals shall have jurisdiction of appeals from all FINAL decisions of district courts.

A final decision is one that ends the litigation

Rule 54(b) - exception to the final judgment rule
In a multi-claim or multi-party action, a judge may direct the entry of a final judgment as to fewer than all the claims or fewer than all the parties
When a judge makes an express determination that there's not just reason for delay and directs entry of fiinal judgment, appeal can be taken from this certified final judgment before entire action disposed of.
District Court judge has discretion whether or not to certify order for appeal
Court of appeals has discretion whether or not to hear the appeal
Appropriate in a case where if there was only one party or one claim in the litigation, would the case be over?


Sussex Drug v. Kanasco (3d Cir. 1990)

Facts: Other litigation going on. Gov't forfeiture proceeding involving same drugs by same manufacturer. There was a partial summary judgment rendered on the issue of liability that the drugs had been adulterated

Sussex drug wants to do what in suit 1: Summary Judgment on its claims against the same two corporate defendants. Breach of warranty and contract

The district court grants the SJ on those claims.
What's left?

Summary Judgment is not a final judgment on the merits

Summary Judgment was rendered on counts 1-4 against corporate defendants seeking $$$ for breach of contract and express/implied warranties

Not included was count 5 for fraud against both individual and corporate defendants
Seeking both compensatory and punitive damages
District court grants Ps request for 54B cert.


Defendants argue that 54B certification is improper
Argue on the merits that d cts use of preclusion was improper
Question - whether district court's order is final judgment as to single claim in multi-claim action

54B certification involves 2 issues:
1. Whether court is dealing with a final judgment in the sense that it's an ultimate disposition of an individual claim in a multi claim action.
2. Whether matter is ready for an appeal

District court has no discretion to certify as "final" that which is not final
Review of "finality" determination is plenary
Readiness for appeal = matter of discretion
Review for abuse of discretion

Court concludes that count 5 (fraud) is basically the same cause of action as counts 1-4 and since with respect to corporate defendants,

Major difference is punitive damages
When liability rests on same transaction or occurrence or series of transactions or occurrences, a count for punitive damages does not mean a separate claim under 54(B)

Here, all P's allegations arise out of single transaction
Count 5 could not have been brought in a separate action - RES JUDICATA -
Court reverses for NO FINAL ORDER
Also reverses for abuse of discretion
Need to review issues currently appealed may be mooted by future developments in suit

Entire recovery P sought could have been awarded under count 5

Use of alternate theory of liability could moot need to review the issue preclusion question currently appealed
If P succeeds in proving fraud, amy not need to review ISSUE PRECLUSION question presented on claims 1-4


Collateral Order Doctrine:
Cohen v. Ben. Ind'l Loan (US 1949)

Case Made Doctrine: Made by US Sup. Ct. in this case
If there are certain orders that are collateral to the merits of the case nad it might be important to review those matters before the case is over, given certain requirements, you can have a certain kind of order appeal under this doctrine:

Order has to be conclusive, final as to that matter
Order must involve important issue that is separate from, collateral to merit
isssue effectively unreviewable on appeal of final judgment.

In the Cohen case, it was a stockholder's derivative action brought in Fed. Ct. in Maine
Under state law, stockholder's bring a stockholders D suit had to post a bond so that if they lost, they could pay expenses of defendant

It was an Eerie question - does fed court apply state law?
The Fed D.Ct said no, the state law would not apply here
Eventually went up to Supreme Court and an appeal was allowed, to the second circuit on this question -
That order they said was separate form, collateral to the merits. It's something that can be reviewed wihotut getting into the merits
It was the judge's final word
Separate from collateral to merits
If you wait until litigation is over and plaintiff's lose and then let the defendant appeal and raise this issue of whether the plaintiffs had to post the bond originally. By then it's too late
So the court says, that's the kind of matter that makes sense to review right away rather than let this case proceed and wait for final judgment.


Cunningham v. Hamilton Co. (1999)
Whether order imposing sanciton on attorney under Rule 37(a)(4) can be immediatley appealed as collateral order

Facts: Attorney essentially flouted all the orders of the magistrate judge with respect to discovery. Didn't answer things on time, etc…

District Cout judge affirmed sanctions order
Defendant's motions to disqualify petitioner as counsel for P due to fact she was material witness in the case
Petitioner took immediate appeal from dct's order affirming the santion.

6th circuit dismissed for want of judisdiction - collateral order doctrine didn't apply

Issues appealed not separate from merits.

S.CT agrees
Attorney's disqualification not relevant
Nonparticipating attorney, like participating attorney, must away final judgment to appeal

Supreme Court reviews final judgment rule and rationale underlying rule
Sanctions order clearly doesn't go under collateral order doctrine

Sanctions order = conclusive
Question about separability of order from merits.

Evaluation of sanctions may require inquiry into importance of info sought, adequacy of responses
Even if sanctions issue were separable from merits, order must be effectively unreviewable on appeal from final judgment

Petitioner argues this si satisfied - ("I'm not an attorney here naymore …. How can I take an appeal_
Generally one who is nonparty to judgment, generally can't appeal
Also argues that sanctions = like contempt orders which are immediately appealable

Court has held that order disqualifying attorney not immediately appealable
Attorneys not treated like other non parties or purposes of appeal
Decision tp appeal must turn on client's interests



Interlocutory Review - FINAL EXCEPTIONS - when the case is not really over
28 USC 1291
(a)(1) : allows appeals as of right form orders granting, continuing, modifying, refusing or dissolving injunctions
(b) : allows trial court to certify for appeal orders that involve controlling question of law as to which there is a substantial ground for difference of opinion where immediate appeal may materially advance the termination of the litigation.



This is a common Civ Pro Law Outline for University Law School classes at:AKRON ALABAMA ALBANY UNION AMERICAN APPALACHIAN *ARIZONA ARIZONA STATE ARKANSAS-FAYETTEVILLE ARKANSAS-LITTLE ROCKAVE MARIA SCHOOL OF LAWBALTIMORE BARRY UNIVERSITY *BAYLOR BOSTON COLLEGE BOSTON UNIVERSITY BRIGHAM YOUNG BROOKLYN CALIFORNIA - BERKELEY CALIFORNIA - DAVIS CALIFORNIA - HASTINGS CALIFORNIA - LOS ANGELES CALIFORNIA WESTERN CAMPBELL CAPITAL CASE WESTERN RESERVE CATHOLIC UNIVERSITY OF AMERICA CHAPMAN CHICAGO CINCINNATI CITY UNIVERSITY OF NEW YORK CLEVELAND STATE COLORADO COLUMBIA CONNECTICUT CORNELL CREIGHTON DAYTON DENVER DePAUL DETROIT MERCY DISTRICT OF COLUMBIA DRAKE DUKE DUQUESNE EMORY FLORIDA FLORIDA A&M*FLORIDA COASTAL FLORIDA INTERNATIONAL*FLORIDA STATE FORDHAM FRANKLIN PIERCE LAW CENTERGEORGE MASONGEORGETOWN GEORGE WASHINGTON GEORGIA GEORGIA STATE GOLDEN GATE +GONZAGA HAMLINE HARVARD HAWAIIHOFSTRAHOUSTONHOWARD IDAHO ILLINOIS ILLINOIS INSTITUTE OF TECHNOLOGY INDIANA - BLOOMINGTON INDIANA - INDIANA POLISINTER-AMERICAN IOWA THE JOHN MARSHALL JOHN MARSHALL (ATLANTA)*JUDGE ADVOCATE GENERAL'S SCHOOLKANSAS KENTUCKY LAVERNE*LIBERTY*LEWIS AND CLARKLOUISIANA STATE LOUISVILLELOYOLA - CHICAGO LOYOLA MARYMOUNT - L.A. LOYOLA - NEW ORLEANSMAINE MARQUETTE MARYLAND MCGEORGE MEMPHIS MERCER MIAMI MICHIGAN STATE MICHIGAN, UNIVERSITY OF MINNESOTA MISSISSIPPI COLLEGE MISSISSIPPI, UNIVERSITY OFMISSOURI-COLUMBIA MISSOURI-KANSAS CITYMONTANANEBRASKANEVADA-Las Vegas NEW ENGLAND NEW MEXICO NEW YORK Law School NEW YORK UNIVERSITYNORTH CAROLINA NORTH CAROLINA CENTRALNORTH DAKOTA NORTHEASTERN NORTHERN ILLINOIS NORTHERN KENTUCKY NORTHWESTERN NOTRE DAME NOVA SOUTHEASTERN OHIO NORTHERN OHIO STATE OKLAHOMA OKLAHOMA CITY OREGONPACE PENNSYLVANIA PENNSYLVANIA STATE PEPPERDINE PITTSBURGHPONTIFICAL CATHOLIC OF P. R.PUERTO RICOQUINNIPIAC REGENT RICHMOND ROGER WILLIAMS RUTGERS - CAMDEN RUTGERS - NEWARK ST. JOHN'S SAINT LOUIS ST. MARY'S ST. THOMAS (FLORIDA) ST. THOMAS (MINNESOTA) SAMFORD SAN DIEGO SAN FRANCISCO SANTA CLARA SEATTLE SETON HALL SOUTH CAROLINA SOUTH DAKOTA SOUTHERN UNIVERSITYSOUTHERN CALIFORNIASOUTHERN ILLINOIS - CARBONDALE SOUTHERN METHODIST SOUTH TEXAS SOUTHWESTERN STANFORD STATE UNIVERSITY OF NEW YORK STETSON SUFFOLK SYRACUSE TEMPLE TENNESSEETEXAS AT AUSTINTEXAS SOUTHERN TEXAS TECH TEXAS WESLEYANTHOMAS JEFFERSON THOMAS M. COOLEYTOLEDOTOURO TULANETULSA UTAHVALPARAISO VANDERBILT VERMONTVILLANOVAVIRGINIAWAKE FORESTWASHBURN WASHINGTON AND LEE WASHINGTON WASHINGTON UNIVERSITYWAYNE STATEWESTERN NEW ENGLAND WESTERN STATE*WEST VIRGINIA WHITTIER +WIDENER WILLAMETTE WILLIAM AND MARY WILLIAM MITCHELL WISCONSIN WYOMING YALE YESHIVA

0 Comments:

Post a Comment

<< Home